Sie sind auf Seite 1von 203

^

N a tio n a l B o a rd o f M e d ic a l E xa m in e rs - G o o g le C h ro m e
i m
. _______________
['
cj
| 0
Q
https://ww w .starttest.eom /api/5.l.1.0/TTDStart.aspx?SVC=lbfd4732-460a-454b-90
b5-9985c0560007
Exam Section 1: Item 1 o f 50 H Mark
National Board o f Medical Examiners Comprehensive Basic Science Self-Assessment
Tim e Remaining: 3 hr 59 min 9 sec
1. A 62-year-old man is brought to the physician by his wife because of increasi
ng confusion during the past 6 months. His wife says that he has become lost twi
ce in the past month when going to work, even though he has been going to the sa
me office for 12 years. She adds that he often has difficulty finding objects su
ch as his glasses and keys, sometimes cannot recall his grandchildren's names, a
nd has become very critical of her cooking, which he used to enjoy. When asked,
he can name only the current president and none of the candidates for an upcomin
g presidential election, although he and his wife watch the television news toge
ther each night. Neurologic examination shows no motor or sensory abnormalities.
His Mini-Mental State Examination score is 19/30. Treatment with a cholinestera
se inhibitor is most likely to improve this patient's memory because of its abil
ity to target synaptic connections between which of the following structures? o
A) The basal forebrain and neurons in the cerebral cortex B) The dentate nucleus
and the thalamus C) The fornix and neurons in the mammillary bodies D) The subs
tantia nigra and the neurons of the globus pallidus O E) The thalamus and neuron
s in layer 4 of the cerebral cortex
Q
https://www.starttest.com/api/5.1.1.0/TTDStart.aspx?SVC=lbfd4732-460a-454b-90b5-
9985c0560007
Exam Section 1: Item 2 o f 50 H Mark
National Board o f Medical Examiners Comprehensive Basic Science Self-Assessment
Tim e Remaining: 3 hr 58 min 52 sec
2. A 25-year-old man has acute lymphocytic leukemia that improves after the admi
nistration of chemotherapy including methotrexate. The beneficial effect of this
drug is most likely the result of inhibition of which of the following enzyme a
ctivities? A) C) Adenosine deaminase Hypoxanthine-guanine phosphoribosyltransfer
ase
o B) DNA polymerase
0D) Thymidylate synthase
C E) Xanthine oxidase
^
N a tio n a l B o a rd o f M e d ic a l E xa m in e rs - G o o g le C h ro m e
i m
. _______________
['
cj
| 0
Q https://www.starttest.eom/api/5.l.1.0/TTDStart.aspx?SVC=lbfd4732-460a-454b-90b
5-9985c0560007 Exam Section 1: Item 3 o f 50 H Mark National Board o f Medical E
xaminers Comprehensive Basic Science Self-Assessment Tim e Remaining: 3 hr 58 mi
n 43 sec
3. A 30-year-old woman comes to the physician for a follow-up examination becaus
e of a 10-year history of recurrent renal calculi. The calculi have primarily be
en composed of calcium oxalate. Physical examination shows no abnormalities. Ser
um calcium and phosphorus concentrations are within the reference ranges. A 24-h
our urine collection shows an increased calcium concentration. The physician rec
ommends that the patient increase her consumption of water and decrease her cons
umption of sodium and protein. The physician also recommends pharmacotherapy wit
h a drug that decreases urinary calcium concentrations. Which of the following i
s the most likely mechanism of action of this drug? A) Decreases 1,25-dihydroxyc
holecalciferol synthesis B) Decreases H+secretion in the distal tubule C) Decrea
ses magnesium reabsorption in the thick ascending limb of the loop of Henle o D)
Increases calcium reabsorption in the distal tubule E) Increases H C 0 3 secret
ion in the proximal tubule F) Increases phosphate reabsorption in the proximal t
ubule G) Inhibits parathyroid hormone secretion
Q
https://www.starttest.com/api/5.1.1.0/TTDStart.aspx?SVC=lbfd4732-460a-454b-90b5-
9985c0560007
Exam Section 1: Item 4 o f 50 H Mark
National Board o f Medical Examiners Comprehensive Basic Science Self-Assessment
Tim e Remaining:
4. A 55-year-old woman undergoing chemotherapy for melanoma develops a temperatu
re of 39C (102.2F) and neutropenia. Her absolute segmented neutrophil count is 500
/mm3. In addition to antibiotics, which of the following substances is most appr
opriate to administer to this patient? o A) Granulocyte-macrophage colony-stimul
ating factor B) lnterleukin-8 (IL-8) CJIL-10 D) Macrophage colony stimulating-fa
ctor E) Stem cell factor (C-kit ligand)
LI
Q
https://www.starttest.com/api/5.1.1.0/TTDStart.aspx?SVC=lbfd4732-460a-454b-90b5-
9985c0560007
Exam Section 1: Item 5 o f 50 H Mark
National Board o f Medical Examiners Comprehensive Basic Science Self-Assessment
Tim e Remaining: 3 hr 58 min 21 sec
5. A 40-year-old woman comes to the physician because of a 2-month history of in
creasing redness around her nose and cheeks, and pimples around her mouth. She a
ppears very upset and tells the physician, "I look like a teenaged Santa Claus."
Which of the following initial responses by the physician is most appropriate?
O A) "Can you tell me how often you go out in the sun and what kind of protectio
n you use?" O B) "Don't worry. Let me assure you this is not a serious condition
." o C) "It's upsetting when you have a skin problem on your face." D) "Maybe a
little. At least you are keeping your sense of humor about the situation." O E)
"You really do look rather odd. I can understand why you are so upset."
LI
^
N a tio n a l B o a rd o f M e d ic a l E xa m in e rs - G o o g le C h ro m e
i m
. _______________
['
cj
| 0
Q https://www.starttest.eom/api/5.l.1.0/TTDStart.aspx?SVC=lbfd4732-460a-454b-90b
5-9985c0560007 Exam Section 1: Item 6 o f 50 H Mark National Board o f Medical E
xaminers Comprehensive Basic Science Self-Assessment Tim e Remaining:
6. A 46-year-old man is brought to the emergency department 2 hours after he fel
l headfirst into the water while attempting to water ski. He says that he has pa
in when he tries to move his left arm. Physical examination shows that the patie
nt is unable to abduct the left upper extremity due to pain. Which of the follow
ing muscles is most likely injured in this patient? A) Infraspinatus B) Subscapu
laris o C) Supraspinatus D) Teres major E) Teres minor
Q
https://www.starttest.com/api/5.1.1.0/TTDStart.aspx?SVC=lbfd4732-460a-454b-90b5-
9985c0560007
Exam Section 1: Item 7 o f 50 0 Mark
National Board o f Medical Examiners Comprehensive Basic Science Self-Assessment
Tim e Remaining: 3 hr 58 min 11 sec
7. A 6-year-old girl is brought to the physician by her mother because of a 2-we
ek history of increased thirst and a 3-kg (6.6-lb) weight loss. Her mother says
that the patient is constantly drinking water. She is at the 75th percentile for
height and 50th percentile for weight. Physical examination shows tachypnea and
dehydration. Laboratory studies show hyperglycemia, metabolic acidosis, and ket
onemia. If a biopsy specimen were obtained from this patient's pancreas, which o
f the following findings in islet cells would now be most likely? A) C) Basement
membrane thickening of capillaries Decrease in mass and deposition of amyloid
o B) Cellular necrosis and lymphocytic infiltration O D) Large (3 cells and nucl
ei E) Marked atrophy and fibrosis
LI
^
N a tio n a l B o a rd o f M e d ic a l E xa m in e rs - G o o g le C h ro m e
i m
. _______________
['
cj
| 0
Q https://www.starttest.eom/api/5.l.1.0/TTDStart.aspx?SVC=lbfd4732-460a-454b-90b
5-9985c0560007 Exam Section 1: Item 8 o f 50 H Mark National Board o f Medical E
xaminers Comprehensive Basic Science Self-Assessment Tim e Remaining: 3 hr 52 mi
n 23 sec
8. A 1-year-old girl is brought to the emergency department because of a 5-day h
istory of vomiting, copious diarrhea, and markedly decreased urinary frequency.
Several other children at the day-care center that she attends have had a simila
r illness during the past week. She appears listless. Physical examination shows
decreased skin turgor, cracked lips, and a dry mouth. The virus identified by E
LISA testing of the stool is resistant to treatment with ether and contains a do
uble-stranded, segmented RNA genome. Which of the following is the most likely c
ausal virus? O A) Adenovirus B) Cytomegalovirus O C) Hepatitis A virus O D) Herp
es simplex virus 1 O E) Influenza virus o F) Rotavirus
^
N a tio n a l B o a rd o f M e d ic a l E xa m in e rs - G o o g le C h ro m e
i m
. _______________
['
cj
| 0
Q https://www.starttest.eom/api/5.l.1.0/TTDStart.aspx?SVC=lbfd4732-460a-454b-90b
5-9985c0560007 Exam Section 1: Item 9 o f 50 H Mark National Board o f Medical E
xaminers Comprehensive Basic Science Self-Assessment Tim e Remaining: 3 hr 52 mi
n 19 sec
9. A 28-year-old woman, gravida 4, para 0, aborta 3, with systemic lupus erythem
atosus comes to the physician at 12 weeks' gestation for an initial prenatal exa
mination. She is concerned that she will not be able to carry her current pregna
ncy to term. The presence of which of the following is most likely increasing th
e risk for complications during pregnancy in this patient? A) Antimitochondrial
antibodies B) Anti neutrophil cytoplasmic antibodies o C) Antiphospholipid antib
odies D) Antistreptolysin 0 antibodies O E) Rheumatoid factor
^
N a tio n a l B o a rd o f M e d ic a l E xa m in e rs - G o o g le C h ro m e
i m
. _______________
['
cj
| 0
Q https://www.starttest.eom/api/5.l.1.0/TTDStart.aspx?SVC=lbfd4732-460a-454b-90b
5-9985c0560007 Exam Section 1: Item 10 o f 50 H Mark National Board o f Medical
Examiners Comprehensive Basic Science Self-Assessment Tim e Remaining: 3 hr 51 m
in 28 sec
10. A study is conducted to assess the accuracy of a new rapid test to detect a
virulent bacterial infection. This infection has an 80% mortality rate if it is
not identified early in its course; however, prompt administration of antibiotic
s decreases the mortality rate to less than 5%. The risks of this antibiotic the
rapy are minimal. A total of 10,000 participants are enrolled and undergo assess
ment with the new test. The graph shows the distribution of infected and noninfe
cted participants according to the results of the test. Which of the following l
abeled points is most appropriate for use as the optimal diagnostic cut point fo
r results of this test?
A)
m B)
C)
D)
E)
Q
https://www.starttest.com/api/5.1.1.0/TTDStart.aspx?SVC=lbfd4732-460a-454b-90b5-
9985c0560007
Exam Section 1: Item 11 o f 50 H Mark
National Board o f Medical Examiners Comprehensive Basic Science Self-Assessment
Tim e Remaining: 3 hr 49 min 0 sec
11. A 60-year-old man comes to the physician because of a 6-month history of dec
reased libido and inability to sustain an erection. He also has had decreased en
ergy during this period and has been falling asleep by 7 pm each evening. He has
not had chronic pain, feelings of depression, or recent psychosocial stressors.
He has had a 2.54-cm (1 -in) loss in height and a 7.62-cm (3-in) increase in wa
ist circumference since his last visit 6 months ago. Physical examination shows
mild gynecomastia and decreased muscle mass throughout. Which of the following i
s the most appropriate pharmacotherapy for this patient? A) Epoetin alfa
0 B) Human growth hormone C) Modafinil
D) Sildenafil o E) Testosterone
LI
Q
https://www.starttest.com/api/5.1.1.0/TTDStart.aspx?SVC=lbfd4732-460a-454b-90b5-
9985c0560007
Exam Section 1: Item 12 o f 50 H Mark
National Board o f Medical Examiners Comprehensive Basic Science Self-Assessment
Tim e Remaining: 3 hr 46 min 22 sec
12.
An 80-year-old woman is brought to the emergency department by police 30 minutes
after she was found wandering in the street, confused and disoriented. She says
that she has no medical conditions and takes no medications. During history tak
ing, she becomes annoyed and agitated, saying, "Stop being so nosey! There's not
hing wrong with me! I feel fine and I just want to go home." She appears frail b
ut alert. Her vital signs are within normal limits. Physical examination shows m
ildly dry mucous membranes. There is no evidence of trauma. She is oriented to p
erson but not to place or time. She is able to provide her home address. Which o
f the following is the most appropriate action by the physician?
o A) Attempt to contact the patient's family to find out more information B) Arr
ange for transportation to take the patient home C) Arrange for the patient to b
e transferred to a skilled nursing care facility D) Administer a sedative-hypnot
ic medication E) Admit the patient to a psychiatric inpatient facility
^
N a tio n a l B o a rd o f M e d ic a l E xa m in e rs - G o o g le C h ro m e
i m
. _______________
['
cj
| 0
Q https://www.starttest.eom/api/5.l.1.0/TTDStart.aspx?SVC=lbfd4732-460a-454b-90b
5-9985c0560007 Exam Section 1: Item 13 o f 50 H Mark National Board o f Medical
Examiners Comprehensive Basic Science Self-Assessment Tim e Remaining: 3 hr 42 m
in 55 sec
A
13. A 25-year-old man comes to the emergency department 5 hours after developing
shortness of breath and chest pain during exercise; he has had no cough or bloo
dy mucus. He has asthma and major depressive disorder. Current medications inclu
de fluticasone inhaler and albuterol. His temperature is 37.1C (98.8F), pulse is 1
10/min, respirations are 30/min, and blood pressure is 90/60 mm Hg. Pulse oximet
ry on room air shows an oxygen saturation of 93%. Cardiac examination shows a no
rmal and S 2with no murmurs and no increase in jugular venous pressure. Laborato
ry studies show: Hemoglobin Hematocrit 13 g/dL 39%
National Board of Medical Examiners - Google Chrom e Q https:/7www.starttest.com
/api/5.1.1.0/ITDStart.aspx?SVC=lbfd4732-460a-454b-90b5-9985c0560007
Exam Section 1: Item 13 o f 50 I Mark
National Board o f Medical Examiners Comprehensive Basic Science Self-Assessment
Tim e Remaining: 3 hr 42 min 44 sec
13. A 25-year-old man comes to the emergency department 5 hours after developing
shortness of breath and chest pain during exercise; he has had no cough or bloo
dy mucus. He has asthma and major depressive disorder. Current medications inclu
de fluticasone inhaler and albuterol. His temperature is 37.1C (98.8F), pulse is 1
10/min, respirations are 30/min, and blood pressure is 90/60 mm Hg. Pulse oximet
ry on room air shows an oxygen saturation of 93%. Cardiac examination shows a no
rmal and S 2with no murmurs and no increase in jugular venous pressure. Laborato
ry studies show: Hemoglobin Hematocrit Arterial blood gas analysis on room air p
H Pco2 Po2 A chest x-ray is shown. Which of the following pulmonary findings are
most likely in this patient? A) Crackles on the left lung base and apex B) Crac
kles on the right lung base o C) Decreased breath sounds on the left C D) Increa
sed wheezes on the left C E) Rhonchi on the right 13 g/dL 39% 7.46 26 mm Hg 60 m
m Hg
Q
https://www.starttest.com/api/5.1.1.0/TTDStart.aspx?SVC=lbfd4732-460a-454b-90b5-
9985c0560007
Exam Section 1: Item 14 o f 50 0 Mark
National Board o f Medical Examiners Comprehensive Basic Science Self-Assessment
Tim e Remaining: 3 hr 40 min 56 sec
14.
A 45-year-old man has a partial colectomy for carcinoma. Which of the following
scenarios indicates the highest likelihood for survival for 5 years after resect
ion of the lesion? A) Moderately differentiated carcinoma invading the musculari
s B) Mucin-producing carcinoma invasive to the serosal surface C) Mucin-producin
g carcinoma metastatic to two regional lymph nodes
o D) Poorly differentiated carcinoma confined to the mucosa E) Well-differentiat
ed carcinoma with hepatic metastasis
LI
^ Q
National
- Google Chrom e
^
https://www.starttest.com/api/5.1.1.0/ITDStart.aspx?SVC=lbfd4732-460a-454b-90b5-
9985c0560007
Exam Section 1: Item 15 o f 50 Mark
National Board o f Medical Examiners Comprehensive Basic Science Self-Assessment
Tim e Remaining: 3 hr 39 min 3 sec
15. A 28-year-old woman undergoes PPD skin testing to determine previous infecti
on with Mycobacterium tuberculosis. Results show a 25-mm, firm area of induratio
n at 48 hours. Analysis of this patient's lesion is most likely to show a predom
inance of which of the following cell types? O A) B lymphocytes o B) Cytotoxic T
lymphocytes O C) Eosinophils ^ D) Macrophages E) Mast cells O F) Neutrophils
Previous
Lab Values

Review
Help -
Pause
O
..in
$
> <
1:44 PM 04/08/2012
^
N a tio n a l B o a rd o f M e d ic a l E xa m in e rs - G o o g le C h ro m e
i m
. _______________
['
cj
| 0
Q https://www.starttest.eom/api/5.l.1.0/TTDStart.aspx?SVC=lbfd4732-460a-454b-90b
5-9985c0560007 Exam Section 1: Item 16 o f 50 H Mark National Board o f Medical
Examiners Comprehensive Basic Science Self-Assessment Tim e Remaining: 3 hr 36 m
in 44 sec
16. A 37-year-old man comes to the emergency department because of a 10-day hist
ory of yellow skin, vague abdominal discomfort, and fatigue. His vital signs are
within normal limits. Physical examination shows generalized icterus. Serum stu
dies show: AST ALT Hepatitis B surface antigen (HBsAg) IgM anti-hepatitis B core
antigen (HBcAg) Anti-hepatitis D virus 320 U/L 340 U/L positive positive negati
ve
Direct damage to infected hepatocytes in this patient is most likely mediated pr
edominantly by which of the following? O A) Antibody against HBcAg B) Antibody a
gainst HBsAg C) Complement via the alternate pathway D) Cytotoxic T lymphocytes
o E) Viral cytopathic effect
^
National Board of Medical Examiners - Google Chrom e
Q https://wvvw.starttest.com/api/5.1.1.0/ITDStart.aspx?SVC=lbfd4732-460a-454b-90
b5-9985c0560007
Exam Section 1: Item 17 o f 50 H Mark
National Board o f Medical Examiners Comprehensive Basic Science Self-Assessment
Tim e Remaining: 3 hr 35 min 46 sec
17. A defect in enzyme activity in amino acid metabolism is induced in an experi
mental animal. Results show an increased serum concentration of phenylalanine an
d decreased concentrations of dopamine and serotonin in the brain. A deficiency
of which of the following is the most likely cause of the findings in this anima
l? O A) Biotin O B) Glutamate C) NADH o D) Tetrahydrobiopterin E) Vitamin B 12(c
obalamin)
Previous
Lab Values

Review
Help -
Pause
O
..in
$
> <
1:47 PM 04/08/2012
National Board of Medical Examiners - Google Chrom e Q https:/7www.starttest.com
/api/5.1.1.0/ITDStart.aspx?SVC=lbfd4732-460a-454b-90b5-9985c0560007
Exam Section 1: Item 18 o f 50 I Mark
National Board o f Medical Examiners Comprehensive Basic Science Self-Assessment
Tim e Remaining: 3 hr 35 min 1 sec
18. A 22-year-old man comes to the physician 1 week after he noticed a painless
ulcer on his penis. Three weeks ago, he had sexual intercourse with a new male p
artner. Physical examination shows regional lymphadenopathy. A photograph of the
penis is shown. Dark-field microscopy of exudate from the ulcer shows a spiral-
shaped organism. Which of the following is the most appropriate pharmacotherapy
for this patient? C A) Cefazolin C B) Ciprofloxacin C C) Erythromycin o D) Penic
illin E) Trimethoprim-sulfamethoxazole
Q
https://www.starttest.com/api/5.1.1.0/TTDStart.aspx?SVC=lbfd4732-460a-454b-90b5-
9985c0560007
Exam Section 1: Item 19 o f 50 0 Mark
National Board o f Medical Examiners Comprehensive Basic Science Self-Assessment
Tim e Remaining: 3 hr 32 min 35 sec
19. A 23-year-old woman comes to the physician for prenatal care. On examination
, she appears euthyroid and the thyroid gland is not palpable. Uterine size is c
onsistent with dates, and fetal pulse is normal. Serum thyroid tests show: Thyro
xine (T4) Thyroid-stimulating hormone 14 pg/dL 2 [jU/mL
Changes in which of the following hormones during pregnancy is primarily respons
ible for the thyroid test results? O A) Androgens O B) Corticosteroids o C) Estr
ogens O D) Progesterones 0 E) Prolactins
LI
^
N a tio n a l
Beard of Medical Examiners - Google Chrome
Q https://www.starttest.com/api/5.1.1.0/ITDStart.aspx?SVC=lbfd4732-460a-454b-90b
5-9985c0560007 Exam Section 1: Item 20 o f 50 Q Mark National Board o f Medical
Examiners Comprehensive Basic Science Self-Assessment Tim e Remaining: 3 hr 29 m
in 27 sec
20. A 56-year-old woman has recently diagnosed carcinoma of the breast. An x-ray
of the chest shows a tumor next to the right side of the heart. An enhanced CT
scan with the tumor invading the pericardium is shown. Which of the following st
ructures is most likely involved? O A) Coronary sinus B) Greater splanchnic vein
o C) Right phrenic nerve O D) Right vagus nerve E) Thoracic duct
Q
https://www.starttest.com/api/5.1.1.0/TTDStart.aspx?SVC=lbfd4732-460a-454b-90b5-
9985c0560007
Exam Section 1: Item 21 o f 50 H Mark
National Board o f Medical Examiners Comprehensive Basic Science Self-Assessment
Tim e Remaining: 3 hr 27 min 25 sec
21.
An 18-month-old girl is brought to the physician by her parents because of physi
cal and mental regression. Pregnancy and delivery were normal. One month after b
irth, the mother noticed a marked startle reaction to noise. At the age of 1 yea
r, she was not able to sit or stand. Physical examination shows macrocephaly. Op
hthalmologic examination shows a cherry-red spot in the macula. Laboratory studi
es in blood leukocytes show a deficiency of (3-hexosaminidase A activity. An exc
essive amount of which of the following metabolites is most likely present in th
is patient's tissues and organs?
O A) Cholesterol B) Dermatan sulfate O C) Glycogen o D) GM2ganglioside E) Sphing
omyelin
^
N a tio n a l B o a rd o f M e d ic a l E xa m in e rs - G o o g le C h ro m e
i m
. _______________
['
cj
| 0
Q https://www.starttest.eom/api/5.l.1.0/TTDStart.aspx?SVC=lbfd4732-460a-454b-90b
5-9985c0560007 Exam Section 1: Item 22 o f 50 H Mark National Board o f Medical
Examiners Comprehensive Basic Science Self-Assessment Tim e Remaining: 3 hr 26 m
in 6 sec
22. A 40-year-old woman with chronic paraplegia caused by multiple sclerosis is
brought to the physician because of severe, debilitating muscle cramps for the p
ast 3 weeks. Treatment with baclofen resolves her muscle cramps. Which of the fo
llowing receptors most likely decreased the muscle spasticity in this patient? O
A) a2 -Adrenoreceptor O B) (32 -Adrenoreceptor o C) Y-Aminobutyric acid Brecept
or O D) Calcium-sensing receptor O E) Ryanodine receptor
^
N a tio n a l B o a rd o f M e d ic a l E xa m in e rs - G o o g le C h ro m e
i m
. _______________
['
cj
| 0
Q https://www.starttest.eom/api/5.l.1.0/TTDStart.aspx?SVC=lbfd4732-460a-454b-90b
5-9985c0560007 Exam Section 1: Item 23 o f 50 H Mark National Board o f Medical
Examiners Comprehensive Basic Science Self-Assessment Tim e Remaining: 3 hr 25 m
in 24 sec
23.
Nitrogen dioxide inhalation in humans causes loss of type I pneumocytes in those
alveoli exposed to high concentrations of the gas. Within 2 weeks, the affected
alveoli are reepithelialized with type I pneumocytes. Which of the following is
the most likely immediate precursor of the new type I pneumocytes? A) Alveolar
macrophages B) Capillary endothelial cells C) Interstitial cells
O D) Type I pneumocytes o E) Type II pneumocytes
LI
Q
https://www.starttest.com/api/5.1.1.0/TTDStart.aspx?SVC=lbfd4732-460a-454b-90b5-
9985c0560007
Exam Section 1: Item 24 o f 50 H Mark
National Board o f Medical Examiners Comprehensive Basic Science Self-Assessment
Tim e Remaining: 3 hr 24 min 23 sec
24. A 72-year-old man has had moderate progressive dementia for the past 3 years
. He has a 25-year history of hypertension. He has a 5-year history of episodic
dysarthria and weakness of the right upper extremity. He has no family history o
f a similar disorder. Which of the following is the most likely diagnosis? O A)
AIDS dementia O B) Dementia, Alzheimer type O C) Huntington disease o D) Multi-i
nfarct (vascular) dementia O E) Parkinsonism
^
National Board of Medical Examiners - Google Chrom e
Q h ttps://w w w .starttest.com /api/5.1 .1.0/TTDStart.aspx?SVC=lbfd4732-460a-45
4b-90b5-9985c0560007
Exam Section 1: Item 25 o f 50 H Mark
National Board o f Medical Examiners Comprehensive Basic Science Self-Assessment
Tim e Remaining: 3 hr 22 min 16 sec
25. A 56-year-old man is brought to the emergency department 1 hour after the su
dden onset of left-sided weakness. Neurologic examination shows weakness of the
lower two thirds of the face on the left, marked weakness of the left upper extr
emity, and moderate weakness of the left lower extremity. Deep tendon reflexes a
re hyperactive in both the upper and lower extremities on the left, but they are
more active in the upper extremity. Normal results of MR angiography are shown.
This patient most likely has an occlusion of which of the following labeled art
eries?
Previous
O
Lab Values
Review

Pause
2:01 PM 04/08/2012

^
National Board of Medical Examiners - Google Chrom e
Q https://www.starttest.com/api/5.1.1.0/ITDStart.aspx?SVC=lbfd4732-460a-454b-90b
5-99S5c0560007
Exam Section 1: Item 25 o f 50 H Mark
National Board o f Medical Examiners Comprehensive Basic Science Self-Assessment
Tim e Remaining: 3 hr 22 min 3 sec
extremities on the left, but they are more active in the upper extremity. Normal
results of MR angiography are shown. This patient most likely has an occlusion
of which of the following labeled arteries?
m A)
B)
C)
D)
E)
Previous
o
Next
Lab Values
Review

Pause
2:01 PM 04/08/2012

m
Q
https://www.starttest.com/api/5.1.1.0/TTDStart.aspx?SVC=lbfd4732-460a-454b-90b5-
9985c0560007
Exam Section 1: Item 26 o f 50 H Mark
National Board o f Medical Examiners Comprehensive Basic Science Self-Assessment
Tim e Remaining: 3 hr 20 min 43 sec
26. A 22-year-old woman comes to the physician because of numbness of her feet a
nd poor balance for 3 months. She follows a vegan diet. She says, "I can't eat ani
mal products, but I do try to keep my proteins and carbohydrates balanced." Phys
ical examination shows absent ankle reflexes. Proprioception and sensation to vi
bration are decreased in both feet. Laboratory studies show a hemoglobin concent
ration of 11 g/dL and mean corpuscular volume of 110 pm 3 This patient most like
ly has a dietary deficiency of which of the following? . A) Folic acid B)lron C)
Vitamin B6(pyridoxine) o D) Vitamin B 12(cobalamin) E) Vitamin C F) Vitamin E
^
N a tio n a l B o a rd o f M e d ic a l E xa m in e rs - G o o g le C h ro m e
i m
. _______________
['
cj
| 0
Q https://www.starttest.eom/api/5.l.1.0/TTDStart.aspx?SVC=lbfd4732-460a-454b-90b
5-9985c0560007 Exam Section 1: Item 27 o f 50 H Mark National Board o f Medical
Examiners Comprehensive Basic Science Self-Assessment Tim e Remaining: 3 hr 19 m
in 49 sec
27.
A 63-year-old man comes to the physician because of weakness and fatigue for 8 m
onths. Physical examination shows massive splenomegaly. Laboratory studies show:
Hemoglobin Hematocrit Leukocyte count Platelet count 8.2 g/dL 25% 6000/mm3 60,0
00/mm3
A peripheral blood smear shows the presence of myelocytes, metamyelocytes, nucle
ated erythrocytes, and teardrop erythrocytes. A bone marrow specimen shows marke
dly thickened trabeculae with replacement of the marrow by cellular fibrous tiss
ue with admixed marrow elements. Which of the following is the most likely diagn
osis? O A) Acute myelogenous leukemia B) Aplastic anemia C) Immune thrombocytope
nic purpura D) Megaloblastic anemia o E) Myelofibrosis
LI
Q
https://www.starttest.com/api/5.1.1.0/TTDStart.aspx?SVC=lbfd4732-460a-454b-90b5-
9985c0560007
Exam Section 1: Item 28 o f 50 H Mark
National Board o f Medical Examiners Comprehensive Basic Science Self-Assessment
Tim e Remaining: 3 hr 17 min 40 sec
28. A 23-year-old primigravid woman at 18 weeks' gestation comes to the physicia
n because she has felt minimal fetal movement during the past 2 weeks. There is
no family history of congenital anomalies. Physical examination shows a uterus c
onsistent in size with an 18-week gestation. Ultrasonography shows decreased amn
iotic fluid and normal-sized fetal kidneys, but the fetal bladder and ureters bi
laterally appear markedly distended. The fetus appears to be male. Which of the
following abnormalities best explains these findings? A) Penile hypospadias B) P
lacental insufficiency C) Polycystic kidney disease o D) Posterior urethral valv
es E) Prostatic nodular hyperplasia
Q
https://www.starttest.com/api/5.1.1.0/TTDStart.aspx?SVC=lbfd4732-460a-454b-90b5-
9985c0560007
Exam Section 1: Item 29 o f 50 H Mark
National Board o f Medical Examiners Comprehensive Basic Science Self-Assessment
Tim e Remaining: 3 hr 16 min 19 sec
29.
A 14-year-old boy collapses while running sprints during high school basketball
tryouts and dies within several minutes. He was considered healthy and had no ab
normal findings on prior physical examinations. His father died at the age of 38
years. At autopsy, the heart weighs 50% more than expected and has marked thick
ening of the left ventricular wall and the interventricular septum. There are no
other gross abnormalities. Which of the following findings is also expected? A)
Diffuse myocardial necrosis
o B) Disorganized hypertrophic cardiac myocytes C) Increased numbers of cardiac
myocytes D) Interstitial infiltration of lymphocytes E) Premature coronary ather
osclerosis
LI
Q
https://www.starttest.com/api/5.1.1.0/TTDStart.aspx?SVC=lbfd4732-460a-454b-90b5-
9985c0560007
Exam Section 1: Item 30 o f 50 H Mark
National Board o f Medical Examiners Comprehensive Basic Science Self-Assessment
Tim e Remaining: 3 hr 14 min 13 sec
30.
During an experiment, an investigator wishes to evaluate the serum cholesterol c
oncentrations of all patients over the age of 50 years on the day of admission t
o the hospital. Both the mean and the median are the same. It is decided to use
the mean as the measure of central tendency. Which of the following is the most
appropriate measure of dispersion to analyze the data for this experiment?
O A) Coefficient of variation B) Interquartile range C) Percentile o D) Range E)
Standard deviation
Q
https:,7www.starttest.com/api/5.1.1.0/ITDStart.aspx?SVC=lbfd4732-460a-454b-90b5-9
985c0560007
Exam Section 1: Item 31 o f 50 I Mark
National Board o f Medical Examiners Comprehensive Basic Science Self-Assessment
Tim e Remaining: 3 hr 13 min 4 sec
31. A 4-year-old boy is brought to the physician because of a 1-day history of r
estlessness, irritability, and intense perirectal itching. He attends a large da
y-care facility. Examination of the perirectal region shows small worms visualiz
ed under bright light. Cellophane tape test results for Enterobius vermicularis
are positive. Which of the following is the most appropriate pharmacotherapy? C
A) Azithromycin o B) Mebendazole C) Metronidazole C D) Pentamidine C E) Tetracyc
line
p 1
Previous Lab Values Review Pause ..ill 2:10 PM 04/08/2012
$
> <
LI
Q
https://www.starttest.com/api/5.1.1.0/TTDStart.aspx?SVC=lbfd4732-460a-454b-90b5-
9985c0560007
Exam Section 1: Item 32 o f 50 H Mark
National Board o f Medical Examiners Comprehensive Basic Science Self-Assessment
Tim e Remaining: 3 hr 10 min 12 sec
32.
A 49-year-old woman comes to the emergency department because of a 3-day history
of fever, shortness of breath, and confusion. She is a postal worker. Her tempe
rature is 38.4C (101.2F), respirations are 28/min, and blood pressure is 100/60 mm
Hg. Physical examination shows nuchal rigidity. Breath sounds are decreased on
the right side of the chest. A lumbar puncture is done. Analysis of cerebrospina
l fluid (CSF) shows: Glucose Total protein Leukocyte count Segmented neutrophils
Monocytes RBC 18mg/dL 138 mg/dL 638/mm3 87% 13% 2300/mm3
A Gram stain of CSF shows large, gram-positive, spore-forming bacilli. A chest x
-ray shows marked widening of the mediastinum. Which of the following extracellu
lar virulence factors most likely enables the causal organism to evade phagocyto
sis? A) Alginate
O B) Glucuronoxylomannan O C) Hyaluronic acid o D) Polyglutamic acid E) Polyribi
tol phosphate
LI
Q
https://www.starttest.com/api/5.1.1.0/TTDStart.aspx?SVC=lbfd4732-460a-454b-90b5-
9985c0560007
Exam Section 1: Item 33 o f 50 H Mark
National Board o f Medical Examiners Comprehensive Basic Science Self-Assessment
Tim e Remaining: 3 hr 9 min 57 sec
33.
A 40-year-old man with alcoholism is admitted to the hospital because of a 2-day
history of confusion. Serum studies show a sodium concentration of 99 mEq/L. He
is treated with 0.9% saline. Four days later, he develops slurred speech. Physi
cal examination shows mild-to-moderate muscle weakness of all extremities and dy
sarthria. Sensation is intact. Babinski sign is present bilaterally. These findi
ngs are most likely caused by a lesion in which of the following locations? A) C
) Bilateral cerebral hemispheres Medial diencephalon
o B) Brain stem O D) Muscle O E) Neuromuscular junction F) Peripheral nerve
LI
^
N a tio n a l B o a rd o f M e d ic a l E xa m in e rs - G o o g le C h ro m e
i m
. _______________
['
cj
| 0
Q https://www.starttest.eom/api/5.l.1.0/TTDStart.aspx?SVC=lbfd4732-460a-454b-90b
5-9985c0560007 Exam Section 1: Item 34 o f 50 H Mark National Board o f Medical
Examiners Comprehensive Basic Science Self-Assessment Tim e Remaining: 3 hr 5 mi
n 41 sec
34. A 3-year-old boy is brought to the physician by his parents because of conce
rns about his ability to heal from injuries. They report that he develops bruise
s and scars with even mild trauma, particularly in comparison to their 5-year-ol
d daughter. He was born at 35 weeks' gestation without complications. He has oth
erwise been healthy and has developed normally. Physical examination shows exten
sible skin, and the joints are freely mobile; there are thin, atrophic scars on
both knees. A photograph of his hands is shown. Which of the following is the mo
st likely cause of this patient's condition? A) Aberrant production of elastin B
) Abnormal synthesis of extracellular glycoprotein o C) Defect in the synthesis
of fibrillar collagen D) Dysfunctional cellular transport protein E) Normal age-
related skin and joint development
Q
https://www.starttest.com/api/5.1.1.0/TTDStart.aspx?SVC=lbfd4732-460a-454b-90b5-
9985c0560007
Exam Section 1: Item 35 o f 50 H Mark
National Board o f Medical Examiners Comprehensive Basic Science Self-Assessment
Tim e Remaining: 3 hr 2 min 16 sec
35.
A randomized cohort study of drug X administered to subjects after a myocardial
infarction found that overall there was no decrease in mortality compared with a
dministration of a placebo after a myocardial infarction. However, on review of
the data, there were statistically fewer deaths among drug X subjects in the sub
group with nontransmural myocardial infarction than in the placebo group. A retr
ospective assessment of the database available for drug X supported the observat
ion. Which of the following is the most appropriate next step? A) Cross-sectiona
l population study of administration of drug X vs. placebo after nontransmural m
yocardial infarction
o B) Prospective, randomized, controlled study of administration of drug X vs. p
lacebo after nontransmural myocardial infarction C) Treatment of all patients wi
th drug X after myocardial infarction D) Treatment of only patients with nontran
smural myocardial infarction with drug X E) Treatment of only patients with tran
smural myocardial infarction with drug X
LI
^
N a tio n a l B o a rd o f M e d ic a l E xa m in e rs - G o o g le C h ro m e
i m
. _______________
['
cj
| 0
Q https://www.starttest.eom/api/5.l.1.0/TTDStart.aspx?SVC=lbfd4732-460a-454b-90b
5-9985c0560007 Exam Section 1: Item 36 o f 50 H Mark National Board o f Medical
Examiners Comprehensive Basic Science Self-Assessment Tim e Remaining: 3 hr 0 mi
n 40 sec
36.
An 88-year-old man loses consciousness when his neck is palpated during a routin
e health maintenance examination. He has a 40-year history of hypertension that
has been well controlled with hydrochlorothiazide and a 20-year history of degen
erative osteoarthritis, for which he occasionally takes anti-inflammatory drugs.
Which of the following is the most likely cause of the syncope?
O A) Complete heart block B) Laryngospasm C) Peripheral arteriole constriction D
) Peripheral arteriole dilation o E) Sinus bradyarrhythmia F) Ventricular tachya
rrhythmia
Q
https://www.starttest.com/api/5.1.1.0/TTDStart.aspx?SVC=lbfd4732-460a-454b-90b5-
9985c0560007
Exam Section 1: Item 37 o f 50 H Mark
National Board o f Medical Examiners Comprehensive Basic Science Self-Assessment
Tim e Remaining: 2 hr 55 min 23 sec
37.
An investigator is planning to create gene therapy for Leigh syndrome, which is
caused by an A ^ G mutation in the mitochondrial tRNALeu gene. Which of the foll
owing is the most likely reason why mitochondria encode their own tRNA?
o A) Mitochondria cannot import proteins or RNA B) Mitochondria produce large am
ounts of reactive oxygen species C) Mitochondria use a non-standard genetic code
D) The unusually high mitochondrial pH denatures nuclear-encoded tRNA E) The un
usually low mitochondrial pH hydrolyzes nuclear-encoded tRNA
^
N a tio n a l B o a rd o f M e d ic a l E xa m in e rs - G o o g le C h ro m e
i m
. _______________
['
cj
| 0
Q https://www.starttest.eom/api/5.l.1.0/TTDStart.aspx?SVC=lbfd4732-460a-454b-90b
5-9985c0560007 Exam Section 1: Item 38 o f 50 H Mark National Board o f Medical
Examiners Comprehensive Basic Science Self-Assessment Tim e Remaining: 2 hr 55 m
in 19 sec
38. A 24-year-old man comes to the physician because of a 3-month history of ina
bility to obtain an erection and occasional penile pain. He also has had mild ch
ronic back pain for 2 years and hemorrhoids that bleed on occasion. He is marrie
d and has a 2-year-old son. He enjoys his job, and he and his wife have a good r
elationship. He lifts weights regularly and trains on his bicycle. He usually ri
des for 2 hours daily during the week and 5 to 6 hours daily on the weekend. On
questioning, he does not recall having spontaneous nocturnal erections. Physical
examination shows no abnormalities except for back tenderness and external hemo
rrhoids. Which of the following is the most likely cause of this patient's erect
ile dysfunction? O A) Compromised blood flow from the pampiniform plexus
e >
B) Damaged blood and nerve supply to the erectile tissue o C) Lumbosacral strain
0 D) Obturator nerve compression 0; E) Pressure of hemorrhoids on the erectile
tissue
LI
? _____________________________
Q https: 7www.starttest.com/api/5.1.1.0/ITDStart.aspx?SVC=lbfd4732-460a-454b-90b
5-9985c0560007 Exam Section 1: Item 39 of 50 I Mark National Board of Medical Ex
aminers Comprehensive Basic Science Self-Assessment Time Remaining: 2 hr 53 min
47 sec
39. A 23-year-old woman at 32 weeks' gestation comes to the emergency department
because of a 1-day history of left flank pain and fever. Her temperature is 39.
1 C (102.3F), pulse is 104/min, respirations are 14/min, and blood pressure is 120
/72 mm Hg. Physical examination shows prominent tenderness over the left costove
rtebral angle. A photomicrograph of a renal biopsy specimen from a similar patie
nt is shown. Which of the following is the most likely diagnosis? o A) Acute pye
lonephritis O B) Acute renal infarction C) Acute tubulointerstitial nephritis D)
Crescentic glomerulonephritis O E) Hemolytic uremic syndrome
r*
s 1' r
A
, r
!
..
-A
Previous
Lab Values
Review
Pause
2:30 PM 04/08/2012
Q
https://www.starttest.com/api/5.1.1.0/TTDStart.aspx?SVC=lbfd4732-460a-454b-90b5-
9985c0560007
Exam Section 1: Item 40 o f 50 H Mark
National Board o f Medical Examiners Comprehensive Basic Science Self-Assessment
Tim e Remaining: 2 hr 51 min 32 sec
40.
A 17-year-old girl is brought to the physician because she has not had a menstru
al period for 15 months. Menarche occurred at the age of 12 years. Menses had oc
curred at regular 28-day intervals until she began a weight-loss regimen 18 mont
hs ago. She is 165 cm (5 ft 5 in) tall and weighs 45 kg (100 lb); BMI is 17 kg/m
2. She says that she eats only 800 calories daily and exercises for more than 3
hours daily. Which of the following serum findings is most likely in this patien
t? A) Decreased circadian variability of free thyroxine concentration
o B) Decreased gonadotropin-releasing hormone pulsatility C) Increased adrenocor
ticotropic hormone pulse frequency D) Increased diurnal variability of follicle-
stimulating hormone concentration E) Reversal of diurnal variability of growth h
ormone concentration
LI
^
N a tio n a l B o a rd o f M e d ic a l E xa m in e rs - G o o g le C h ro m e
i m
. _______________
['
cj
| 0
Q
https://ww w .starttest.eom /api/5.l.1.0/TTDStart.aspx?SVC=lbfd4732-460a-454b-90
b5-9985c0560007
Exam Section 1: Item 41 o f 50 H Mark
National Board o f Medical Examiners Comprehensive Basic Science Self-Assessment
Tim e Remaining: 2 hr 51 min 29 sec
41. A 55-year-old woman with congestive heart failure has progressive shortness
of breath and chest pain. An x-ray of the chest shows bilateral pleural effusion
s. Laboratory studies done on pleural fluid and serum show:
Pleural Fluid
Glucose Protein Lactate dehydrogenase Total nucleated cell count Which of the fo
llowing is the most likely cause of the pleural effusion? A) Decreased lymphatic
drainage B) Decreased oncotic pressure C) Decreased serum protein concentration
o D) Increased hydrostatic pressure E) Increased vascular permeability 80 mg/dL
2 g/dL 25 U/L 500/mm3
Serum
100 mg/dL 7 g/dL 50 U/L
^
N a tio n a l B o a rd o f M e d ic a l E xa m in e rs - G o o g le C h ro m e
i m
. _______________
['
cj
| 0
Q
https://ww w .starttest.eom /api/5.l.1.0/TTDStart.aspx?SVC=lbfd4732-460a-454b-90
b5-9985c0560007
Exam Section 1: Item 42 o f 50 H Mark
National Board o f Medical Examiners Comprehensive Basic Science Self-Assessment
Tim e Remaining: 2 hr 49 min 49 sec
42.
One day after a total abdominal hysterectomy with salpingo-oophorectomy for stag
e III cervical cancer, a 42-year-old woman has abdominal distention. She has pas
sed only 200 ml_ of urine since the operation. Ultrasonography shows an accumula
tion of fluid in the abdominal cavity; the physician suspects that the fluid is
urine. This patient most likely sustained injury to the ureter during intraopera
tive ligation of which of the following arteries?
o A) Internal iliac artery B) Ovarian artery C) Pudendal artery C D) Superior ve
sicle arteries E) Ureteric branches of the renal artery
^
National Board of Medical Examiners - Google Chrom e
Q https://www.starttest.com/api/5.1.1.0/ITDStart.aspx?SVC=lbfd4732-460a-454b-90b
5-9985c0560007 Exam Section 1: Item 43 o f 50 H Mark National Board o f Medical
Examiners Comprehensive Basic Science Self-Assessment Tim e Remaining: 2 hr 48 m
in 53 sec
43. During an experiment using adult rats, bone formation and resorption are stu
died. The addition of compound X causes a decrease in bone resorption. Compound
X is most likely which of the following? o A) Calcitonin O B) Parathyroid hormon
e C) Prednisone D) Thyroxine (T4) E) Tumor necrosis factor
Previous
Lab Values

Review
Help -
Pause
O
..ill
$
'< >
2:36 PM 04/08/2012
Q
https://www.starttest.com/api/5.1.1.0/TTDStart.aspx?SVC=lbfd4732-460a-454b-90b5-
9985c0560007
Exam Section 1: Item 44 o f 50 H Mark
National Board o f Medical Examiners Comprehensive Basic Science Self-Assessment
Tim e Remaining: 2 hr 46 min 43 sec
44.
An 18-year-old woman sustains second-degree burns over her right thigh and lower
leg after spilling hot grease while working at a fast-food restaurant. Six week
s later, the surface area of the wound has markedly decreased. Which of the foll
owing processes best explains the wound contracture in this patient? A) Cross-li
nking of type II collagen fibers B) Inhibition of stromelysin-1 (matrix metallop
roteinase-3)
o C) Myofibroblast activity O D) P-selectin activity E) Respiratory burst in mac
rophages
LI
Q
https://www.starttest.com/api/5.1.1.0/TTDStart.aspx?SVC=lbfd4732-460a-454b-90b5-
9985c0560007
Exam Section 1: Item 45 o f 50 H Mark
National Board o f Medical Examiners Comprehensive Basic Science Self-Assessment
Tim e Remaining: 2 hr 45 min 21 sec
45.
Osteogenesis imperfecta (01) is an autosomal dominant disorder characterized cli
nically by a remarkably wide variety of manifestations that includes fragile bon
es that fracture easily, blue sclerae, hearing loss, hyperextensible joints, and
enamel dysplasia. Which of the following is the most likely explanation for the
pleiotropic effects of the 01 gene?
O A) Contiguous gene deletion o B) Expression of a defective gene in multiple ti
ssues O C) Loss of imprinting at multiple loci D) Somatic loss of heterozygosity
in alleles E) Variable trinucleotide expansion
Q
https://www.starttest.com/api/5.1.1.0/TTDStart.aspx?SVC=lbfd4732-460a-454b-90b5-
9985c0560007
Exam Section 1: Item 46 o f 50 H Mark
National Board o f Medical Examiners Comprehensive Basic Science Self-Assessment
Tim e Remaining: 2 hr 44 min 25 sec
46. A healthy 28-year-old woman delivers a female newborn at term with a cranial
malformation. There is no family history of similar findings. Physical examinat
ion of the newborn shows a large defect in the calvaria and meninges with a prim
itive brain. The newborn dies 2 days later. In this patient's future pregnancies
, an increased maternal serum concentration of which of the following during the
second trimester will most likely indicate the same malformation? o A) a-Fetopr
otein O B) Free 3-human chorionic gonadotropin C) Inhibin A D) Pregnancy-associa
ted plasma protein A O E) Unconjugated estriol
LI
^
N a tio n a l B o a rd o f M e d ic a l E xa m in e rs - G o o g le C h ro m e
i m
. _______________
['
cj
| 0
Q
https://ww w .starttest.eom /api/5.l.1.0/TTDStart.aspx?SVC=lbfd4732-460a-454b-90
b5-9985c0560007
Exam Section 1: Item 47 o f 50 H Mark
National Board o f Medical Examiners Comprehensive Basic Science Self-Assessment
Tim e Remaining: 2 hr 43 min 41 sec
47. A 2-month-old boy is brought to the physician for a follow-up examination. H
e was delivered at term to a 27-year-old woman with myasthenia gravis. Immediate
ly after delivery, he developed respiratory distress that lasted for 2 weeks. Ph
ysical examination today shows no abnormalities. The most likely cause of this p
atient's transient disease is transplacental transfer of which of the following
antibody types? A) IgA B)lgD O C) IgE <> D) IgG E)lgM
^
National Beard of Medical Examiners - Google Chrome Q https://www.starttest.com/
api/5.1.1.0/ITDStart.aspx?SVC=lbfd4732-460a-454b-90b5-9985c0560007 Exam Section
1: Item 48 o f 50 | l^ ark National Board o f Medical Examiners Comprehensive Ba
sic Science Self-Assessment Tim e Remaining: 2 hr 42 min 13 sec
48. A 1-year-old boy is brought to the physician for a well-child examination. P
hysical examination shows pallor. Laboratory studies show: Hemoglobin Hematocrit
Erythrocyte count Mean corpuscular volume Reticulocyte count Leukocyte count Pl
atelet count 8.5 g/dL 26% 2.1 million/mm3 65 |jm 3(N=70-86) 0.5% 8000/mm3with a
normal differential 480,000/mm3
'O
g^
co

o
V : ^ 3
A peripheral blood smear is shown in the photomicrograph. Which of the following
is the most likely diagnosis? A) Acute lymphoblastic leukemia B) Folic acid def
iciency o C) Iron deficiency anemia D) Sickle cell disease E) Vitamin B 12 (coba
lamin) deficiency
0 - n C % < L 0
J r \ 0
X
J
'
loda o&j
u o
^
N a tio n a l B o a rd o f M e d ic a l E xa m in e rs - G o o g le C h ro m e
i m
. _______________
['
cj
| 0
Q https://www.starttest.eom/api/5.l.1.0/TTDStart.aspx?SVC=lbfd4732-460a-454b-90b
5-9985c0560007 Exam Section 1: Item 49 o f 50 H Mark National Board o f Medical
Examiners Comprehensive Basic Science Self-Assessment Tim e Remaining: 2 hr 40 m
in 23 sec
49.
A 32-year-old woman comes to the physician because of intermittent abdominal cra
mps and diarrhea, alternating with constipation. The stools are loose and brown;
there is no blood or mucus. She has had these symptoms for 15 years, but they h
ave become more frequent during the past 3 months. She has not had fever or weig
ht loss. Colonoscopy 3 years ago showed no abnormalities. She recently was promo
ted to a management position at her company. She is 160 cm (5 ft 3 in) tall and
weighs 75 kg (165 lb); BMI is 29 kg/m2. Her temperature is 37C (98.6F), pulse is 7
2/min, respirations are 16/min, and blood pressure is 130/76 mm Hg. Examination
shows no abnormalities. Test of the stool for occult blood is negative. Which of
the following is the most likely diagnosis?
A) Colon cancer B) D) Inflammatory bowel disease Pancreatic insufficiency o C) I
rritable bowel syndrome O E) Peptic ulcer disease
LI
^
National Board of Medical Examiners - Google Chrom e
Q h ttps://w w w .starttest.com /api/5.1 .1.0/TTDStart.aspx?SVC=lbfd4732-460a-45
4b-90b5-9985c0560007
Exam Section 1: Item 50 o f 50 H Mark
National Board o f Medical Examiners Comprehensive Basic Science Self-Assessment
Tim e Remaining: 2 hr 39 min 42 sec
50. A 45-year-old man with recently diagnosed early-onset Parkinson disease come
s to the physician for a follow-up examination. He has a long history of major d
epressive disorder successfully treated with fluoxetine. Treatment with which of
the following antiparkinsonian drugs is contraindicated in this patient? A) Ama
ntadine C B) Benztropine C)Levodopa C D) Ropinirole o E) Selegiline
Previous
O
Lab Values
Review

Pause
2:45 PM 04/08/2012

^
N a tio n a l B o a rd o f M e d ic a l E xa m in e rs - G o o g le C h ro m e
i m
. _______________
['
cj
| 0
Q
https://ww w .starttest.eom /api/5.l.1.0/TTDStart.aspx?SVC=lbfd4732-460a-454b-90
b5-9985c0560007
Exam Section 2: Item 1 o f 50 H Mark
National Board o f Medical Examiners Comprehensive Basic Science Self-Assessment
Tim e Remaining: 4 hr 17 min 30 sec
1. A 35-year-old woman comes to the emergency department because of abdominal cr
amps, nausea, and vomiting for 8 hours. Her pulse is 106/min and regular, and bl
ood pressure is 140/96 mm Hg. Physical examination shows rhinorrhea, excessive l
acrimation, and diaphoresis. There is piloerection over most of the skin. The pu
pils are dilated, equal in size, and responsive to direct and indirect light. Th
ere is a resting tremor involving both upper extremities. During the examination
, the patient is restless and yawns constantly. This patient is most likely expe
riencing withdrawal symptoms from which of the following substances? A) Cannabis
0 B) Cocaine O C) Diazepam O D) Ethanol o E) Heroin 0 F) Phenobarbital
Q
https://www.starttest.com/api/5.1.1.0/TTDStart.aspx?SVC=lbfd4732-460a-454b-90b5-
9985c0560007
Exam Section 2: Item 2 o f 50 H Mark
National Board o f Medical Examiners Comprehensive Basic Science Self-Assessment
Tim e Remaining: 4 hr 16 min 16 sec
2. A 58-year-old man from Eastern Europe who has recently immigrated to the USA
with his family comes to the physician with his wife for an initial physical exa
mination. Moderately severe emphysema is diagnosed, and continuous oxygen treatm
ent is recommended. One week later, the wife calls the physician and tells him t
hat her husband refuses to use the oxygen treatment. He is scheduled for a follo
w-up visit the next day. Which of the following is the most appropriate initial
action for the physician to take during the next follow-up visit? A) Determine w
hether the patient is competent to make an informed decision B) Inform the patie
nt about the deleterious effects of his decision o C) Initiate a discussion abou
t the patient's concerns regarding the treatment D) Recognize that the couple is
undergoing many adjustments and stresses and wait 6 months before discussing hi
s noncompliance E) Recognize that the patient may be wary of the treatment due t
o his cultural background and enlist the aid of community members
LI
Q
https://www.starttest.com/api/5.1.1.0/TTDStart.aspx?SVC=lbfd4732-460a-454b-90b5-
9985c0560007
Exam Section 2: Item 3 o f 50 H Mark
National Board o f Medical Examiners Comprehensive Basic Science Self-Assessment
Tim e Remaining: 4 hr 15 min 24 sec
3. A 46-year-old man falls three stories from an open window and fractures the s
haft of his right femur and pelvis. Thirty-six hours after stabilization of the
fractures, he becomes confused and develops dyspnea, tachypnea, and arterial hyp
oxemia. Examination shows petechiae in the conjunctivae and over the pectoral re
gions. An x-ray of the chest shows a diffuse alveolar infiltrate. Pulmonary lava
ge shows cells staining for oil red 0 . Which of the following is the most likel
y cause of the respiratory dysfunction? A) Bacterial pneumonia B) Disseminated i
ntravascular coagulation o C) Fat embolism D) Myocardial infarction O E) Pulmona
ry thromboembolism
LI
^
N a tio n a l B o a rd o f M e d ic a l E xa m in e rs - G o o g le C h ro m e
i m
. _______________
['
cj
| 0
Q https://www.starttest.eom/api/5.l.1.0/TTDStart.aspx?SVC=lbfd4732-460a-454b-90b
5-9985c0560007 Exam Section 2: Item 4 o f 50 H Mark National Board o f Medical E
xaminers Comprehensive Basic Science Self-Assessment Tim e Remaining: 4 hr 13 mi
n 28 sec
4. A 63-year-old man comes to the physician because of a 3-month history of abdo
minal pain that is only partially relieved by antacids. He states that he has al
so tried fasting, but it has had little to no effect on the pain. Palpation of t
he abdomen shows exquisite tenderness focused to the right of the midline. Labor
atory studies show a fasting serum gastrin concentration of 800 pg/mL (N<100) an
d no evidence of Helicobacter pylori infection. Secretin infusion after the pati
ent has fasted increases serum gastrin concentration to 1150 pg/mL. Secretion of
which of the following is also most likely increased in this patient as the res
ult of this infusion? O A) Ghrelin O B) Glucagon-like peptide-1 O C) Hepatic bil
e secretion o D) Pancreatic bicarbonate E) Saliva
^
N a tio n a l B o a rd o f M e d ic a l E xa m in e rs - G o o g le C h ro m e
i m
. _______________
['
cj
| 0
Q https://www.starttest.eom/api/5.l.1.0/TTDStart.aspx?SVC=lbfd4732-460a-454b-90b
5-9985c0560007 Exam Section 2: Item 5 o f 50 H Mark National Board o f Medical E
xaminers Comprehensive Basic Science Self-Assessment Tim e Remaining: 4 hr 10 mi
n 1 sec
5. A 25-year-old man with type 1 diabetes mellitus comes to the physician becaus
e of poor diabetic control for 3 months. He has had ketones in his urine, and hi
s blood glucose concentrations have been in the 200-400 mg/dl_ range, particular
ly after exercise. He has been receiving two injections of insulin daily. Physic
al examination shows no abnormalities. His hemoglobin A 1c is 12%. Following exe
rcise on a treadmill for 45 minutes, the patient has an increase in his serum gl
ucose concentration from 175 m g/dLto 225 mg/dl_. Which of the following hepatic
enzymes is most likely activated initially by exercise in this patient? O A) De
branching enzyme O B) Glycogen synthase C) Phospholipase C o D) Phosphorylase ki
nase O E) Protein kinase B F) Protein kinase G G) Receptor protein-tyrosine kina
se
^
N a tio n a l B o a rd o f M e d ic a l E xa m in e rs - G o o g le C h ro m e
i m
. _______________
['
cj
| 0
Q https://www.starttest.eom/api/5.l.1.0/TTDStart.aspx?SVC=lbfd4732-460a-454b-90b
5-9985c0560007 Exam Section 2: Item 6 o f 50 H Mark National Board o f Medical E
xaminers Comprehensive Basic Science Self-Assessment Tim e Remaining: 4 hr 7 min
34 sec
6.
A 1-year-old boy is brought to the physician by his mother because of failure to
thrive. He has had intermittent respiratory tract infections and diarrhea durin
g the past 6 months. He was born at term following an uncomplicated pregnancy. H
e has received only diluted cow's milk and a homemade gruel of rice, sugar, and
water since the age of 3 months. His vital signs are normal. He is at the 30th p
ercentile for weight. He appears apathetic. Physical examination shows dry skin,
moderate hepatomegaly, a distended abdomen, an umbilical hernia protruding 2 cm
, and 2+ pitting edema of the lower back and face. There is no jaundice. His hai
r is easily plucked. Laboratory studies show pancytopenia. Which of the followin
g is the most likely cause of his symptoms?
O A) Cystic fibrosis B) Enterobacterial infection C) Extrahepatic biliary atresi
a D) Hepatolenticular degeneration (Wilson disease) o E) Protein-calorie malnutr
ition
Jp National Board of Medical Examiners - Google Chrom e g https: 7 www.starttest
.corn/api/5.1.1.0/ITDStart.aspx?SVC=lbfd4732-460a-454b-90b5-9985c0560007
Exam Section 2: Item 7 o f 50 0 Mark
National Board o f Medical Examiners Comprehensive Basic Science Self-Assessment
Tim e Remaining: 4 hr 0 min 29 sec
7. A 66-year-old man with type 2 diabetes mellitus and hypertension is brought t
o the emergency department 30 minutes after the sudden onset of left eyelid droo
ping, double vision, and mild weakness of the right hand and leg. His pulse is 8
8/min and regular, and blood pressure is 159/99 mm Hg. Examination of the head s
hows a substantially droopy left eyelid, and in primary gaze, the left eye is ex
otropic and somewhat lower than the right. He has slowed finger movements on the
right, a pronator drift with the right hand, and mild hyperreflexia on the righ
t. Which of the following is the most likely site and diagnosis of the patient's
lesion? A) Angular gyrus (Gerstmann syndrome) B) Dorsolateral thalamus (Dejerin
e-Roussy syndrome) C) Lower medulla (Wallenberg syndrome) o D) Lower midbrain (W
eber syndrome) O E) Upper spinal cord (Horner syndrome)
Previous
Lab Values

Review

Help
I*
-
Pause
O
..in
Tb
r ~ r :
$
> <
3:11 PM 04/08/2012
National Board of Medical Examiners - Google Chrom e Q https://wvw.starttest.com
/api/511M TDStart.aspx7SVC3lbfd4732-460a-454b-90b5-9985c0560007
I___

I
Exam Section 2: Item 8 o f 50 I Mark
National Board o f Medical Examiners Comprehensive Basic Science Self-Assessment
Tim e Remaining: 3 hr 59 min 35 sec
8. A study is conducted to assess the relationship between age and serum total c
holesterol concentration in three groups. Group X consists of 50 children (ages
6 to 10 years), Group Y consists of 50 adolescents (ages 12 to 16 years), and Gr
oup Z consists of 50 young adults (ages 21 to 25 years). The fasting serum total
cholesterol concentration is measured in all participants. Which of the followi
ng statistical tests is most appropriate to compare the serum total cholesterol
concentration results among Groups X, Y, and Z in this study? = > A) Analysis of
variance o B) Chi-square test 0 C) Independent West D) Multiple logistic regres
sion E) Paired West C F) Pearson correlation
[^f National Board of Medical Examiners - Google Chrom e
Q https://www.starttest.com/api/5.1.1.0/TTDStart.aspx?SVC=lbfd4732-460a-454b-90b
5-9985c0560007 Exam Section 2: Item 9 o f 50 H Mark National Board o f Medical E
xaminers Comprehensive Basic Science Self-Assessment Tim e Remaining: 3 hr 56 mi
n 41 sec
9. The graph shows the response elicited by different concentrations of drug X i
n a system containing spare receptors in the absence (solid curve) and presence
(dashed curves) of two different concentrations of drug Y. Drug Y alone has no e
ffect. Which of the following best describes drug Y? A) Competitive reversible a
ntagonist
O B) Full agonist O C) Inverse agonist = > D) Noncompetitive antagonist o E) Par
tial agonist
[Drug X] (pM) [Drug Y] = 0 uM [Drug Y] = 1 uM [Drug Y] = 10 pM
Previous
O
Lab Values
Review

Pause
3:15 PM 04/08/2012

[^f National Board of Medical Examiners - Google Chrom e Q https://www.starttest


.com/api/5.1.1.0/TTDStart.aspx?SVC=lbfd4732-460a-454b-90b5-9985c0560007
Exam Section 2: Item 10 o f 50 I Mark
National Board o f Medical Examiners Comprehensive Basic Science Self-Assessment
Tim e Remaining: 3 hr 54 min 47 sec
10. A 72-year-old woman has a persistent headache, weakness of the lower extremi
ties, and increased intracranial pressure. Cerebral arteriography shows a neopla
sm causing inferior displacement of the artery at the location indicated by the
arrows shown. Which of the following is the most likely displaced artery? o A) A
nterior cerebral B) Basilar O C) Middle cerebral O D) Posterior cerebral O E) Ve
rtebral
p 1
Lab Values Review
3:17 PM 04/08/2012
National Board of Medical Examiners - Google Chrom e Q https://www.starttest.com
/api/5.1.1.0/ITDStart.aspx?SVC=lbfd4732-460a-454b-90b5-9985c0560007
I___

I
Exam Section 2: Item 11 o f 50 0 Mark
National Board o f Medical Examiners Comprehensive Basic Science Self-Assessment
Tim e Remaining: 3 hr 52 min 51 sec
11. A 24-year-old man is brought to the physician by his wife because of a 2-day
history of progressive confusion. His temperature is 38.3C (101 F). Physical exam
ination shows no other abnormalities. When he speaks, he enunciates clearly, and
his phrasing is of average length. However, he uses words and word-like utteran
ces in a manner that makes little sense. He does not follow any commands. Which
of the following is the most likely diagnosis? A) Cerebral toxoplasmosis B) Herp
es simplex encephalitis O C) HIV encephalopathy o D) Meningococcal meningitis O
E) Subdural empyema
[^p National Board of Medical Examiners - Google Chrom e Q https://www.starttest
.com/api/5.1.1.0/TTDStart.aspx?SVC=lbfd4732-460a-454b-90b5-9985c0560007
I'
Exam Section 2: Item 12 o f 50 I Mark
National Board o f Medical Examiners Comprehensive Basic Science Self-Assessment
Tim e Remaining: 3 hr 50 min 26 sec
12.
During an experiment with laser scanning microscopy, it is observed that bone ma
rrow-derived stem cells migrate through the thymus. Which of the following is mo
st likely to occur as a result of this process?
O A) CD40 induction O B) Germinal center formation C) Membrane immunoglobulin ex
pression D) Thymic involution o E) T-lymphocyte antigen receptor selection
Previous
Lab Values

Review
Help -
Pause
O
.,il, $ > <
3:22 PM 04/08/2012
National Board of Medical Examiners - Google Chrom e Q https://wvw.starttest.com
/api/511M TDStart.aspx7SVC3lbfd4732-460a-454b-90b5-9985c0560007
I___

I
Exam Section 2: Item 13 o f 50 I Mark
National Board o f Medical Examiners Comprehensive Basic Science Self-Assessment
Tim e Remaining: 3 hr 49 min 21 sec
13.
A 43-year-old woman with systemic lupus erythematosus has nephrotic syndrome. Ex
amination of renal tissue obtained on biopsy shows a diffuse proliferative glome
rulonephritis with electron-dense deposits along the glomerular basement membran
e. Immunofluorescent studies show granular deposits of complement along the base
ment membrane. Which of the following is the most likely cause of these findings
? A) Antibodies directed against glomerular basement membrane B) Antibodies dire
cted against viral antigens expressed on endothelial cells
o C) Anti-DNA/DNA immune complex deposition in glomeruli D) Autoantibodies to po
docyte antigens E) CD8+ T-lymphocyte cytotoxicity of virus-infected mesangial ce
lls
[^p National Board of Medical Examiners - Google Chrom e Q https://ww w .startte
st.eom /api/5.l.1.0/TTDStart.aspx?SVC=lbfd4732-460a-454b-90b5-9985c0560007
Exam Section 2: Item 14 o f 50 I Mark
National Board o f Medical Examiners Comprehensive Basic Science Self-Assessment
Tim e Remaining: 3 hr 48 min 28 sec
14. A 52-year-old man with a family history of cardiovascular disease begins tre
atment with 81-mg aspirin for cardioprotection. This protective effect is best e
xplained by which of the following actions of this drug on platelets? A) Irrever
sible inhibition of prostacyclin (PGI2) production B) Irreversible inhibition of
prostaglandin E2(PGE2) production o C) Irreversible inhibition of thromboxane A
2 (TXA2) production O D) Reversible inhibition of PGE2 production O E) Reversib
le inhibition of PGI2 production 0 F) Reversible inhibition of TXA2 production
Previous
Lab Values

Review
Help -
Pause
O
..in
$
> <
3:24 PM 04/08/2012
National Board of Medical Examiners - Google Chrom e Q https://www.starttest.com
/api/5.1.1.0/ITDStart.aspx?SVC=lbfd4732-460a-454b-90b5-9985c0560007
Exam Section 2: Item 15 o f 50 I Mark
National Board o f Medical Examiners Comprehensive Basic Science Self-Assessment
Tim e Remaining: 3 hr 47 min 33 sec
15. A 50-year-old man is brought to the emergency department 45 minutes after he
was found unresponsive at home. He has a 3-day history of progressive lethargy
and increasingly severe headache. He hit his head in a fall 4 weeks ago; at that
time, he did not seek medical attention. He appears stuporous. His pulse is 60/
min, and blood pressure is 150/90 mm Hg. Neurologic examination shows right hemi
paresis. A CT scan of the head without contrast is shown. The most likely cause
of this patient's condition is bleeding from which of the following structures?
A) Arteriovenous malformation
C B) Berry aneurysm o C) Bridging cortical vein D) Middle meningeal artery E) Su
perior sagittal sinus
[^p National Board of Medical Examiners - Google Chrom e Q https://www.starttest
.com/api/5.1.1.0/TTDStart.aspx?SVC=lbfd4732-460a-454b-90b5-9985c0560007
I'
Exam Section 2: Item 16 o f 50 0 Mark
National Board o f Medical Examiners Comprehensive Basic Science Self-Assessment
Tim e Remaining: 3 hr 46 min 30 sec
16. A 63-year-old woman with an 18-year history of type 2 diabetes mellitus has
decreased visual acuity. Funduscopic examination of both eyes shows hemorrhages
and "cotton wool" spots. Which of the following is the predominant underlying pa
thologic condition in this disease? O A) Atherosclerosis o B) Microangiopathy O
C) Optic neuritis O D) Thrombosis O E) Vasculitis
Previous
Lab Values

Review
Help
-
Pause

O
.,il,
$
3:26 PM
> <
04/08/2012
National Board of Medical Examiners - Google Chrom e Q https://www.starttest.com
/api/5.1.1.0/ITDStart.aspx?SVC=lbfd4732-460a-454b-90b5-9985c0560007
I___

I
Exam Section 2: Item 17 o f 50 I Mark
National Board o f Medical Examiners Comprehensive Basic Science Self-Assessment
Tim e Remaining: 3 hr 45 min 32 sec
17. A previously healthy 29-year-old man is brought to theemergency department 1
hour after colliding with a barrier while snowboarding.Physicalexaminationshows
edema and deformity of the right upper extremity. An x-ray of the affectedextrem
ity shows adisplaced fracture of the surgical neck of the humerus. Thispatient i
s most likely to have impaired or absent sensation in which of the following loc
ations ipsilateral to the fracture? o A) Lateral aspect of the arm B) Medial asp
ect of the arm C) Anterior aspect of the mid forearm D) Posterior aspect of the
mid forearm 0 E) Pad of the thumb 0 F) Pad of the ring finger
[^p National Board of Medical Examiners - Google Chrom e Q https://www.starttest
.com/api/5.1.1.0/TTDStart.aspx?SVC=lbfd4732-460a-454b-90b5-9985c0560007
I'
Exam Section 2: Item 18 o f 50 I Mark
National Board o f Medical Examiners Comprehensive Basic Science Self-Assessment
Tim e Remaining: 3 hr 42 min 45 sec
18. An investigator is developing a new drug, Drug X, to protect health care wor
kers after accidental inoculation with blood containing HIV. The drug is designe
d to block viral entry into the cell. Which of the following is the most appropr
iate target for Drug X? A) CD8 o B) Chemokine receptor 0 C) Fc receptor
0 D) HIV protease 0 E)lntegrase 0 F) lnterleukin-2 (IL-2) receptor G) Reverse tr
anscriptase 0 H) Tat protein
Previous
Lab Values

Review
Help -
Pause
O
..in
$
> <
3:30 PM 04/08/2012
[^f National Board of Medical Examiners - Google Chrom e
1
r
*
-I
0
https://www.starttest.com/api/5.1.1.0/ITDStart.aspx?SVC=lbfd4732-460a-454b-90b5-
9985c0560007
Exam Section 2: Item 19 o f 50 H Mark
National Board o f Medical Examiners Comprehensive Basic Science Self-Assessment
Tim e Remaining: 3 hr 40 min 48 sec
19. A 59-year-old man comes to the physician because of a 3-day history of yello
w-tinted eyes and intermittent regurgitation of small amounts of blood. He has h
ad progressive fatigue and increased abdominal girth during the past month. Phys
ical examination shows scleral icterus, pale conjunctivae, and a protuberant abd
omen. An abdominal fluid wave is palpated. Laboratory studies show: Hemoglobin H
ematocrit Mean corpuscular hemoglobin concentration Mean corpuscular volume Seru
m Alkaline phosphatase AST ALT 9.8 g/dL 29% 28% Hb/cell 70 pm3 210 U/L 20 U/L 10
U/L
..... r . ^ *
vV y-> r.'^ ' A * ],,.
P
. :
' *'% - /?., *
' >
4:
'\8rMsAE K ' .: j** :**
*7.
*
s'
i.
i * 4
j
i
t* '
/* jc'-r
k * 4
a
'S
< *
I ^ p *^
`
*
A biopsy specimen of tissue from the liver is shown in the photomicrograph. Uppe
r gastrointestinal endoscopy is most likely to show which of the following? o A)
Esophageal varices B) Hemorrhagic gastritis C) Hiatal hernia D) Mallory-Weiss l
aceration O E) Schatzki ring
;
V .f
m m tm A '.E
Previous
O
p 1
Lab Values
Review -

Pause
C l
.,il, ^ > < 3:32 PM 04/08/2012
LI
National Board of Medical Examiners - Google Chrom e Q https://www.starttest.com
/api/5.1.1.0/ITDStart.aspx?SVC=lbfd4732-460a-454b-90b5-9985c0560007
I___

I
Exam Section 2: Item 20 o f 50 0 Mark
National Board o f Medical Examiners Comprehensive Basic Science Self-Assessment
Tim e Remaining: 3 hr 36 min 5 sec
20. A 28-year-old woman wants to lose weight. She is 160 cm (5 ft 3 in) tall and
weighs 81.5 kg (180 lb); BMI is 32 kg/m2. She consumes 1800 calories daily and
has a sedentary life-style. Assume that there are 3500 calories per pound and th
at brisk walking consumes 500 calories per hour. Which of the following regimens
is most likely to help her lose 0.9 kg (2 lb) weekly?
A) B) C) D) E) F)
m
Calories Per Day Decrease by 100 Decrease by 100 Decrease by 500 Decrease by 500
No change No change
Brisk Walking Daily
1 hour /4 1 hour 1 hour /4 1 hour 1 hour /4 1 hour
National Board of Medical Examiners - Google Chrom e Q https://www.starttest.com
/api/5.1.1.0/ITDStart.aspx?SVC=lbfd4732-460a-454b-90b5-9985c0560007
I___

I
Exam Section 2: Item 21 o f 50 I Mark
National Board o f Medical Examiners Comprehensive Basic Science Self-Assessment
Tim e Remaining: 3 hr 34 min 43 sec
21. A 20-year-old man who is an army recruit comes to the physician because of a
n 8-hour history of headache, stiff neck, and rash. His temperature is 40C (104F),
pulse is 120/min, respirations are 25/min, and blood pressure is 90/50 mm Hg. P
hysical examination shows nuchal rigidity and a petechial rash over the extremit
ies. A lumbar puncture is done. Culture of cerebrospinal fluid grows gram-negati
ve diplococci. Which of the following would be used to further classify this cau
sal organism into a serogroup? o A) Antibodies to capsular polysaccharide B) Com
parison of aerobic and anaerobic growth C) Hemolytic pattern on sheep blood agar
D) Measurement of total genome size O E) Phage typing
National Board of Medical Examiners - Google Chrom e Q https://www.starttest.com
/api/5.1.1.0/ITDStart.aspx?SVC=lbfd4732-460a-454b-90b5-9985c0560007
Exam Section 2: Item 22 o f 50 I Mark
National Board o f Medical Examiners Comprehensive Basic Science Self-Assessment
Tim e Remaining: 3 hr 32 min 56 sec
22. A 17-year-old boy is brought to the emergency department because of a high-g
rade fever and increasingly severe diffuse abdominal pain during the past 3 hour
s. The pain started 2 days ago, but it resolved without medical treatment until
hours prior to admission, when the abdominal pain recurred. He appears anxious,
pale, and diaphoretic. His temperature is 38C (100.4F). Abdominal examination show
s guarding and rebound tenderness. An exploratory operation is done. A sample of
fluid obtained from the abdominal cavity is found to have a specific gravity gr
eater than 1.020, numerous leukocytes (mainly segmented neutrophils), and cellul
ar debris. Which of the following best describes this fluid? A) Ascites B) Blood
C) Exudate D) Lymphedema o E) Transudate
National Board of Medical Examiners - Google Chrom e Q https://www.starttest.com
/api/5.1.1.0/ITDStart.aspx?SVC=lbfd4732-460a-454b-90b5-9985c0560007
I___

I
Exam Section 2: Item 23 o f 50 I Mark
National Board o f Medical Examiners Comprehensive Basic Science Self-Assessment
Tim e Remaining: 3 hr 31 min 17 sec
23. A 41-year-old man comes to the physician's office because of recurrent heada
ches, flushing, and palpitations for the past 3 months. Blood pressure measureme
nts have ranged from 130/76 mm Hg to 200/120 mm Hg. CT scan of the abdomen shows
a right suprarenal mass. An increase in the serum concentration of which of the
following hormones is the most likely cause of the episodic hypertension in thi
s patient? O A) Aldosterone
0 B) Angiotensin II 0 C) Cortisol 0 D) Endothelin
o E) Epinephrine
Q
https://www.starttest.com/api/5.1.1.0/TTDStart.aspx?SVC=lbfd4732-460a-454b-90b5-
9985c0560007
Exam Section 2: Item 24 o f 50 H Mark
National Board o f Medical Examiners Comprehensive Basic Science Self-Assessment
Tim e Remaining: 3 hr 29 min 11 sec
24. Two programs for the treatment of patients with newly detected hypercholeste
rolemia were tried in a community. Program A was used in one district of the com
munity, and Program B was used in another. After four years, 45% of the 2200 pat
ients on Program A and 49% of the 1900 patients on Program B had been successful
ly treated for hypercholesterolemia. The difference between the success rates fo
r the two programs was statistically significant (p < .01). Health officials, ho
wever, decided not to change to Program B in the first district because the magn
itude of the difference was so small. Which of the following best explains their
decision? o A) They attributed the difference in success rates to chance alone
B {> B) They distinguished between statistical significance and practical import
ance of the difference in success rates D) They felt the samples were too small
to justify a decision in favor of Program B C) They felt the p value was too sma
ll to justify a decision in favor of Program B
LI
National Board of Medical Examiners - Google Chrom e Q https://www.starttest.com
/api/5.1.1.0/ITDStart.aspx?SVC=lbfd4732-460a-454b-90b5-9985c0560007
I___

I
Exam Section 2: Item 25 o f 50 I Mark
National Board o f Medical Examiners Comprehensive Basic Science Self-Assessment
Tim e Remaining: 3 hr 28 min 26 sec
25.
A 22-year-old woman comes to the physician because of a 5-day history of white s
pots over her upper torso. Two days ago, she returned from a vacation at the bea
ch, where she noticed that certain areas of her chest and back did not tan. Her
vital signs are within normal limits. Physical examination shows 2-cm, hypopigme
nted macules with areas of fine scaling over the chest, back, and proximal upper
extremities. Microscopic examination of a scraping from one of the macules show
s hyphae and yeast forms. Which of the following is the most likely causal organ
ism? A) Candida albicans B) Cladosporium cladosporioides
o C) Maiassezia furfur
G D) Microsporum canis E) Sporothrix schenckii
National Board of Medical Examiners - Google Chrom e Q https://www.starttest.com
/api/5.1.1.0/ITDStart.aspx?SVC=lbfd4732-460a-454b-90b5-9985c0560007
I___

I
Exam Section 2: Item 26 o f 50 0 Mark
National Board o f Medical Examiners Comprehensive Basic Science Self-Assessment
Tim e Remaining: 3 hr 26 min 46 sec
26. A 50-year-old man comes to the physician for a follow-up examination. He has
been receiving nortriptyline twice daily for major depressive disorder. The dos
age of his medication was doubled from once to twice daily 2 months ago because
the initial dosage was ineffective. At this visit, the patient tells the physici
an that his symptoms have not improved. Mental status examination shows a flat a
ffect. Laboratory studies show a plasma nortriptyline concentration 30% of the l
ower end of the normal range. Nortriptyline is eliminated mainly by hepatic meta
bolism catalyzed by the cytochrome P450 2D6 (CYP2D6). Which of the following bes
t explains the patient's lack of clinical response? O A) Induction of CYP2D6 by
nortriptyline o B) Inheritance of an amplified CYP2D6 locus C) Inheritance of tw
o inactive CYP2D6 alleles O D) Inhibition of CYP2D6 by nortriptyline E) Pharmaco
dynamic tolerance
[^p National Board of Medical Examiners - Google Chrom e Q https://www.starttest
.com/api/5.1.1.0/TTDStart.aspx?SVC=lbfd4732-460a-454b-90b5-9985c0560007
I'
Exam Section 2: Item 27 o f 50 I Mark
National Board o f Medical Examiners Comprehensive Basic Science Self-Assessment
Tim e Remaining: 3 hr 25 min 4 sec
27. A 41 -year-old woman is brought to the emergency department by her husband b
ecause of pelvic pain for 6 hours. She also has had heavy menstrual periods duri
ng the past 3 months. A CT scan of the abdomen shows a tumor in the left ovary.
An oophorectomy is scheduled. During the procedure, which of the following struc
tures immediately deep to the infundibulopelvic (suspensory) ligament must be pr
otected while isolating and ligating the ovarian vessels in this ligament? O A)
Hypogastric nerve 0 B) Lumbosacral trunk 0 C) Round ligament D) Sympathetic trun
k o E) Ureter
Previous
Lab Values

Review
Help
-
Pause

O
.,il,
$
3:48 PM
> <
04/08/2012
National Board of Medical Examiners - Google Chrom e Q https://www.starttest.com
/api/5.1.1.0/ITDStart.aspx?SVC=lbfd4732-460a-454b-90b5-9985c0560007
I___

I
Exam Section 2: Item 29 o f 50 I Mark
National Board o f Medical Examiners Comprehensive Basic Science Self-Assessment
Tim e Remaining: 3 hr 21 min 32 sec
29. A female newborn is delivered at 34 weeks' gestation. Her temperature is 37.
4C (99.3F), pulse is 96/min, respirations are 20/min, and blood pressure is 108/72
mm Hg. Physical examination shows microcephaly and bilateral cataracts; there i
s no cyanosis. There is a continuous "machinery-like" murmur increasing in inten
sity during systole and waning during diastole; it is loudest in the second left
intercostal space and radiates down the left sternal border. The apical impulse
is prominent, and there is a thrill in the second left intercostal space. The p
hysician suspects that the findings in this patient were caused by a congenital
infection. Which of the following infectious agents is the most likely cause? O
A) Cytomegalovirus O B) Epstein-Barr virus O C) Herpes simplex virus D) HIV
Mycobacterium tuberculosis o F) Rubella virus
E) O G) Toxoplasma gondii H) Treponema pallidum
National Board of Medical Examiners - Google Chrom e Q https://www.starttest.com
/api/5.1.1.0/ITDStart.aspx?SVC=lbfd4732-460a-454b-90b5-9985c0560007
Exam Section 2: Item 29 o f 50 I Mark
National Board o f Medical Examiners Comprehensive Basic Science Self-Assessment
Tim e Remaining: 3 hr 20 min 16 sec
29. A female newborn is delivered at 34 weeks' gestation. Her temperature is 37.
4C (99.3F), pulse is 96/min, respirations are 20/min, and blood pressure is 108/72
mm Hg. Physical examination shows microcephaly and bilateral cataracts; there i
s no cyanosis. There is a continuous "machinery-like" murmur increasing in inten
sity during systole and waning during diastole; it is loudest in the second left
intercostal space and radiates down the left sternal border. The apical impulse
is prominent, and there is a thrill in the second left intercostal space. The p
hysician suspects that the findings in this patient were caused by a congenital
infection. Which of the following infectious agents is the most likely cause? O
A) Cytomegalovirus O B) Epstein-Barr virus O C) Herpes simplex virus D) HIV
Mycobacterium tuberculosis o F) Rubella virus
E) O G) Toxoplasma gondii H) Treponema pallidum
National Board of Medical Examiners - Google Chrom e Q https://www.starttest.com
/api/5.1.1.0/ITDStart.aspx?SVC=lbfd4732-460a-454b-90b5-9985c0560007
Exam Section 2: Item 30 o f 50 0 Mark
National Board o f Medical Examiners Comprehensive Basic Science Self-Assessment
Tim e Remaining: 3 hr 18 min 13 sec
30.
A 5-year-old girl is brought to the physician because of a 1-year history of gen
eralized tonic-clonic seizures that occur more frequently when she has a fever.
Her 3-year-old brother has had similar episodes. Neurologic examination shows no
abnormalities. Genetic testing shows a mutation affecting one of the subunits o
f the Y-aminobutyric acidA receptor. Which of the following mechanisms is the mo
st likely cause of seizures in this patient? A) Decreased postsynaptic calcium i
nflux
o B) Decreased postsynaptic chloride influx C) Decreased presynaptic calcium inf
lux D) Increased postsynaptic potassium influx E) Increased presynaptic chloride
influx F) Increased presynaptic potassium influx
N a tio n a l B e a rd o f M e d ic a l E xa m in e rs - G o o g le C h ro m e
Q
https://ww w .starttest.eom /api/5.l.1.0/TTDStart.aspx?SVC=lbfd4732-460a-454b-90
b5-9985c0560007
Exam Section 2: Item 31 o f 50 H Mark
National Board o f Medical Examiners Comprehensive Basic Science Self-Assessment
(g/ease Wait
31.
A 63-year-old man with alcoholism is diagnosed with hepatic encephalopathy. Trea
tment with lactulose therapy via nasogastric tube is begun. The effectiveness of
this intervention requires which of the following mechanisms? A) Absorbed lactu
lose binding alcohol in the blood, decreasing blood alcohol concentrations B) La
ctulose binding of ingested alcohol within the colonic lumen for excretion C) Me
tabolism to glucose and galactose, increasing serum glucose concentrations D) Os
motic diarrhea, flushing out ingested alcohol from the gut
o E) Trapping of ammonia in the colon by acidic metabolites of lactulose
National Board of Medical Examiners - Google Chrom e Q https://www.starttest.com
/api/5.1.1.0/ITDStart.aspx?SVC=lbfd4732-460a-454b-90b5-9985c0560007
I___

I
Exam Section 2: Item 32 o f 50 I Mark
National Board o f Medical Examiners Comprehensive Basic Science Self-Assessment
Tim e Remaining: 3 hr 15 min 59 sec
32.
During a 5-year study at a hospital, data about antimicrobial drug use for respi
ratory infections and antimicrobial susceptibility for respiratory isolates are
compiled. At the end of 5 years, the data show that levofloxacin was the most co
mmon drug used to treat respiratory infections and that its usage increased sign
ificantly since the beginning of the study. Resistance of Klebsiella pneumoniae
to levofloxacin increased from 20% to 54% during the 5-year period. Based on the
se data, there is a greater likelihood of detecting which of the following in th
e latter isolates compared with the bacteria recovered at the beginning of the s
tudy? A) Expression of AmpC 3-lactamases B) Expression of carbapenemase C) Expre
ssion of extended-spectrum (3-lactamases D) Mutation of the gene encoding dihydr
ofolate reductase E) Mutation of the gene encoding elongation factor-2
o F) Mutation of the gene encoding topoisomerase II
National Board of Medical Examiners - Google Chrom e Q https://www.starttest.com
/api/5.1.1.0/ITDStart.aspx?SVC=lbfd4732-460a-454b-90b5-9985c0560007
I___

I
Exam Section 2: Item 33 o f 50 I Mark
National Board o f Medical Examiners Comprehensive Basic Science Self-Assessment
Tim e Remaining: 3 hr 15 min 21 sec
33. A 32-year-old man sustains a head injury in a motorcycle accident and is adm
itted to the intensive care unit. Six days later he becomes confused and is very
thirsty. His serum sodium concentration is 158 mEq/L, and his urine output is 6
.1 L/day. These findings most likely indicate that he has a disorder that is aff
ecting which of the following labeled parts of the nephron shown?
National Board of Medical Examiners - Google Chrom e Q https://wvw.starttest.com
/api/511M TDStart.aspx7SVC3lbfd4732-460a-454b-90b5-9985c0560007
I___

I
Exam Section 2: Item 34 o f 50 I Mark
National Board o f Medical Examiners Comprehensive Basic Science Self-Assessment
Tim e Remaining: 3 hr 14 min 32 sec
34. A 70-year-old man is admitted to the hospital because of painless jaundice f
or the past 2 weeks. He has had dark urine and white stools for the past 7 days.
Examination of the abdomen discloses no abnormalities. A CT scan of the abdomen
shows a large poorly defined soft tissue density in the head of the pancreas. W
hich of the following is the most likely cause of the jaundice? A) Cholelithiasi
s o B) Common bile duct obstruction O C) Liver metastases O D) Pancreatic duct o
bstruction E) Porta hepatis metastases
National Board of Medical Examiners - Google Chrom e Q https://www.starttest.com
/api/5.1.1.0/ITDStart.aspx?SVC=lbfd4732-460a-454b-90b5-9985c0560007
I___

I
Exam Section 2: Item 35 o f 50 0 Mark
National Board o f Medical Examiners Comprehensive Basic Science Self-Assessment
Tim e Remaining: 3 hr 12 min 48 sec
35. A 75-year-old man comes to the physician because of a 1-month history of leg
swelling. His respirations are 20/min. Physical examination shows jugular venou
s distention. There is 3+ pitting edema of the lower extremities below the knees
. The lungs are clear to auscultation. Echocardiography is most likely to show a
n enlargement of which of the following vascular structures? A) C) Inferior vena
cava and left atrium Right atrium and left atrium
O B) Left atrium and left ventricle o D) Right atrium and right ventricle O E) R
ight ventricle and left atrium
National Board of Medical Examiners - Google Chrom e Q https://www.starttest.com
/api/5.1.1.0/ITDStart.aspx?SVC=lbfd4732-460a-454b-90b5-9985c0560007
Exam Section 2: Item 36 o f 50 0 Mark
National Board o f Medical Examiners Comprehensive Basic Science Self-Assessment
Tim e Remaining: 3 hr 10 min 54 sec
36.
A 10-year-old boyis brought to the physicianby his parents for a follow-up exami
nation because of mental retardation and pigmentary anomalies. Heunderwent an op
eration at the age of 3 years to correctsyndactyly between the middle and ring f
ingers bilaterally. Physical examinationshows streaky hyperpigmentation. Chromos
omal analysis shows 46,XY in 15 cells and 69,XXY in 5 cells. Which of the follow
ing is the most likely explanation for the karyotype findings in this patient?
C A) Deletion o B) Duplication C) Inversion D) Mosaicism E) Ring chromosome F) T
ranslocation
[^p National Board of Medical Examiners - Google Chrom e Q https://ww w .startte
st.eom /api/5.l.1.0/TTDStart.aspx?SVC=lbfd4732-460a-454b-90b5-9985c0560007
I'
Exam Section 2: Item 37 o f 50 I Mark
National Board o f Medical Examiners Comprehensive Basic Science Self-Assessment
Tim e Remaining: 3 hr 9 min 49 sec
37. A 20-year-old woman comes to the physician for preconceptional counseling be
cause she is concerned that she will not be able to conceive children. She has n
ever had a menstrual period. She has normal female body habitus. Serum follicle-
stimulating hormone concentration is 120 mlU/mL. Her karyotype is 45,X. In couns
eling this patient, which of the following should be recommended? o A) Oocyte do
nation O B) Ovulation induction O C) Surrogacy with her eggs O D) Timed intercou
rse
Previous
Lab Values

Review
Help
-
Pause

O
.,il,
$
4:04 PM
> <
04/08/2012
[^p National Board of Medical Examiners - Google Chrom e Q https://www.starttest
.com/api/5.1.1.0/TTDStart.aspx?SVC=lbfd4732-460a-454b-90b5-9985c0560007
I'
Exam Section 2: Item 38 o f 50 I Mark
National Board o f Medical Examiners Comprehensive Basic Science Self-Assessment
Tim e Remaining: 3 hr 9 min 17 sec
38. Alveolar ventilation and C 0 2 production double during moderate exercise in
a 48-year-old man. Which of the following best describes the effect on arterial
Pco2? O A) Divided by 4 O B) Divided by 2 o C) Not changed D) Doubled E) Quadru
pled
Previous
Lab Values

Review
Help -
Pause
O
..in
$
> <
4:04 PM 04/08/2012
National Board of Medical Examiners - Google Chrom e g https://www.starttest.eom
/api/5.1.1.0/TTDStart.aspx?SVC=lbfd4732-460a-454b-90b5-9985c0560007
Exam Section 2: Item 39 o f 50 0 Mark
National Board o f Medical Examiners Comprehensive Basic Science Self-Assessment
Tim e Remaining: 3 hr 8 min 12 sec
39. A 56-year-old woman with hypertension has Raynaud disease. Which of the foll
owing anti hypertensive drugs is most beneficial for both disorders? O A) Captop
ril O B)Clonidine O C) Losartan o D) Nifedipine O E) Propranolol
National Board of Medical Examiners - Google Chrom e Q https://www.starttest.com
/api/5.1.1.0/ITDStart.aspx?SVC=lbfd4732-460a-454b-90b5-9985c0560007
I___

I
Exam Section 2: Item 40 o f 50 I Mark
National Board o f Medical Examiners Comprehensive Basic Science Self-Assessment
Tim e Remaining: 3 hr 6 min 59 sec
40. A 1-year-old boy is brought to the emergency department because of a 6-day h
istory of temperatures to 39.4C (103F) and a 2-day history of a severe diaper rash
and swelling of his hands and feet. His temperature is now 39.4C (103F). A photog
raph of the genital area is shown. The face and lips appear red and the conjunct
ivae appear injected. There is bilateral cervical lymphadenopathy. The lungs are
clear to auscultation. Cardiac examination shows an S 3 gallop with no murmur.
There is edema and erythema of the hands and feet. Which of the following pathol
ogic findings is most likely in this patient? o A) Acute arteritis with aneurysm
s in coronary arteries B) Epidermal hyperplasia with epidermal microabscesses an
d parakeratosis C) Granulomas with caseous necrosis in cervical lymph nodes D) G
ranulomatous arteritis in cervical and temporal arteries E) Paracortical lymphoi
d hyperplasia with eosinophilic intranuclear inclusions in perihilar lymph nodes
[^f National Board of Medical Examiners - Google Chrom e
Q https://www.starttest.com/api/5.1.1.0/TTDStart.aspx?SVC=lbfd4732-460a-454b-90b
5-9985c0560007 Exam Section 2: Item 41 o f 50 Q Mark National Board o f Medical
Examiners Comprehensive Basic Science Self-Assessment Tim e Remaining: 3 hr 4 mi
n 19 sec
41. A 26-year-old man comes to the physician because of fever, cough, chest pain
, and malaise for 2 weeks. He moved to central California 6 months ago. A comple
te blood count shows mild eosinophilia. A chest x-ray shows patchy bronchopneumo
nia. Culture of the sputum grows a mold with the morphology shown in the photomi
crograph, with the arrow indicating the infectious particle. Which of the follow
ing is the most likely causal organism? A)
Actinomyces israelii
B) Coccidioides immitis
C) Histoplasma capsulatum D) Legionella pneumophila E) Mycobacterium tuberculosi
s F) Nocardia brasiliensis G) Staphylococcus aureus H) Streptococcus pneumoniae
Previous
o
Lab Values
Review

Pause
4:10 PM 04/08/2012

National Board of Medical Examiners - Google Chrom e


|. C = 3
| 0
0
https://www.starttest.com/api/5.1.1.0/ITDStart.aspx?SVC=lbfd4732-460a-454b-90b5-
9985c0560007
Exam Section 2: Item 42 o f 50 H Mark
National Board o f Medical Examiners Comprehensive Basic Science Self-Assessment
Tim e Remaining: 3 hr 2 min 27 sec
42. A 37-year-old man comes to the physician because of progressive weakness and
diarrhea during the past 2 months. He has had a 15-kg (33-lb) weight loss due t
o a decrease in appetite during this period. His blood pressure is 65/40 mm Hg.
Physical examination shows hyperpigmentation of the skin. Laboratory studies sho
w hyponatremia and hyperkalemia. Hypotension and electrolyte disturbances in thi
s patient are most likely due to the absence of a hormone that is primarily synt
hesized in which of the following labeled regions in the photomicrograph of the
organ shown?
Previous
O
Lab Values
Review

Pause
C l
4:11 PM 04/08/2012
Q
https://www.starttest.com/api/5.1.1.0/TTDStart.aspx?SVC=lbfd4732-460a-454b-90b5-
9985c0560007
Exam Section 2: Item 43 o f 50 H Mark
National Board o f Medical Examiners Comprehensive Basic Science Self-Assessment
Tim e Remaining: 2 hr 58 min 28 sec
43. A previously healthy 55-year-old woman comes to the physician because of the
gradual onset of fever, fatigue, and pain in her muscles and joints during the
past 3 weeks; she has had a 3.6-kg (8-lb) weight loss during this period. She do
es not smoke cigarettes, drink alcohol, or use illicit drugs. She takes no medic
ations. Her temperature is 38.2C (100.8F), pulse is 90/min, and blood pressure is
140/95 mm Hg. Examination of the trunk and extremities shows areas of raised, re
ticular, cyanotic discoloration consistent with livedo reticularis. There is lef
t footdrop. Laboratory studies show: Erythrocyte sedimentation rate Urine Blood
Protein RBC casts 110 mm/h 2+ 2+ present
Serum test results are positive for perinuclear anti neutrophil cytoplasmic anti
bodies. Which of the following is the most likely diagnosis? A) Allergic interst
itial nephritis B) Antiphospholipid antibody syndrome O C) IgA nephropathy O D)
Occult neoplasm o E) Vasculitis
LI
National Board of Medical Examiners - Google Chrom e Q https://www.starttest.com
/api/5.1.1.0/ITDStart.aspx?SVC=lbfd4732-460a-454b-90b5-9985c0560007
I___

I
Exam Section 2: Item 44 o f 50 I Mark
National Board o f Medical Examiners Comprehensive Basic Science Self-Assessment
Tim e Remaining: 2 hr 57 min 14 sec
44. A 5-year-old girl is brought to the physician by her parents because she is
developing breasts. Examination shows breast enlargement and scant pubic hair. X
-rays of the left hand show a bone age of 9 years. An intravenous bolus of gonad
otropin-releasing hormone induces a marked increase in serum luteinizing hormone
concentration 1 hour later. Which of the following drugs is most appropriate to
suppress premature puberty in this patient? O A) ACTH O B) Hydrocortisone C) Ke
toconazole o D) Leuprolide O E) Progesterone O F) Spironolactone O G) Tamoxifen
O H) Testosterone
National Board of Medical Examiners - Google Chrom e Q https://www.starttest.com
/api/5.1.1.0/ITDStart.aspx?SVC=lbfd4732-460a-454b-90b5-9985c0560007
I___

I
Exam Section 2: Item 45 o f 50 I Mark
National Board o f Medical Examiners Comprehensive Basic Science Self-Assessment
Tim e Remaining: 2 hr 55 min 3 sec
45. A 27-year-old man is brought to the emergency department by his mother 30 mi
nutes after a generalized tonic-clonic seizure. She reports that he is now becom
ing more arousable. He has schizophrenia, and his mother says that he stopped ta
king his haloperidol 3 months ago. He has been drinking a lot of water during th
e past 2 months to "rid himself of toxins." On arrival, he is initially unrespon
sive but then becomes arousable. His temperature is 37.8C (100F), pulse is 90/min
and regular, and blood pressure is 110/78 mm Hg. Examination shows normal skin t
urgor. The neck is supple with normal range of motion. Cardiopulmonary examinati
on shows no abnormalities. Abdominal examination shows no abnormalities. There i
s no peripheral edema. Laboratory studies show: Serum Na+ K+ Clh c o 3-
Urea nitrogen Creatinine Urine specific gravity
116 mEq/L 4.2 mEq/L 80 mEq/L 18 mEq/L 8 mg/dL 0.8 mg/dL 1.002
Urine dipstick shows no other abnormalities. Results of additional laboratory st
udies are pending. Which of the following is the most likely cause of this patie
nt's hyponatremia? O A) Adverse effect of haloperidol B) Hypothyroidism o C) Psy
chogenic polydipsia D) Syndrome of inappropriate secretion of ADH (vasopressin)
O E) Tumor in the central nervous system
National Board of Medical Examiners - Google Chrom e Q https://www.starttest.com
/api/5.1.1.0/ITDStart.aspx?SVC=lbfd4732-460a-454b-90b5-9985c0560007
I___

I
Exam Section 2: Item 46 o f 50 0 Mark
National Board o f Medical Examiners Comprehensive Basic Science Self-Assessment
Tim e Remaining: 2 hr 53 min 27 sec
46.
A study is conducted of the oxygen-binding properties of hemoglobin in an experi
mental animal. The hydrogen ion concentration is increased in blood. Results sho
w that the oxygenhemoglobin dissociation curve shifts to the right. This effect
will most likely result in which of the following processes? A) Binding of carbo
n dioxide in tissue capillaries
O B) Binding of oxygen in the lungs C) Decrease in erythrocyte 2,3-bisphosphogly
cerate (2,3-BPG) concentration D) Increase in erythrocyte 2,3-BPG concentration
E) Release of carbon dioxide in the lungs o F) Release of oxygen in tissue capil
laries
National Board of Medical Examiners - Google Chrom e Q https://www.starttest.com
/api/5.1.1.0/ITDStart.aspx?SVC=lbfd4732-460a-454b-90b5-9985c0560007
Exam Section 2: Item 47 o f 50 I Mark
National Board o f Medical Examiners Comprehensive Basic Science Self-Assessment
Tim e Remaining: 2 hr 51 min 42 sec
47. A 20-year-old man with mental retardation is brought to the physician for a
routine examination. Physical examination shows a long face, prominent ears, and
large testes. His younger brother and a maternal uncle have similar features. A
nalysis of the patient's DNA shows 800 CGG repeated sequences (N<60) in the 5' u
ntranslated region of the FM R 1 gene. This region of expanded CGG trinucleotide
repeats is heavily methylated. Which of the following is the most likely effect
of these expanded nucleotide repeats on transcription of FM R1 mRNA? C A) Alter
ation of mRNA splicing B) Decreased transcription C) Enhancement of mRNA degrada
tion o D) Incorporation of CGG repeats into mRNA E) Increased binding of RNA pol
ymerase
National Board of Medical Examiners - Google Chrom e Q https://www.starttest.com
/api/5.1.1.0/ITDStart.aspx?SVC=lbfd4732-460a-454b-90b5-9985c0560007
I___

I
Exam Section 2: Item 48 o f 50 I Mark
National Board o f Medical Examiners Comprehensive Basic Science Self-Assessment
Tim e Remaining: 2 hr 50 min 33 sec
48. A 54-year-old woman undergoes a biopsy of a 5-cm coin lesion in the right up
per lobe of the lung. Examination of the biopsy specimen shows a proliferation o
f irregularly shaped glands containing cells with hyperchromatic and pleomorphic
nuclei invading the pulmonary parenchyma and lymphovascular spaces. Which of th
e following best describes this patient's pulmonary lesion? o A) Adenocarcinoma
O B) Carcinoid tumor C) Hamartoma D) Small cell carcinoma E) Squamous cell carci
noma
[^p National Board of Medical Examiners - Google Chrom e Q https://ww w .startte
st.eom /api/5.l.1.0/TTDStart.aspx?SVC=lbfd4732-460a-454b-90b5-9985c0560007
Exam Section 2: Item 49 o f 50 0 Mark
National Board o f Medical Examiners Comprehensive Basic Science Self-Assessment
Tim e Remaining: 2 hr 48 min 4 sec
49. A healthy 40-year-old man comes to the physician requesting a medication to
help him sleep on an overnight flight to Australia. Physical examination shows n
o abnormalities. Which of the following is the most appropriate pharmacotherapy
for this patient? O A) Bupropion O B) Buspirone C) Clonazepam O D) Flurazepam o
E) Zolpidem
Previous
O
Next
O

H r %
Lab Values
Review

Help
Pause

&
r
i
T
O
1
-
..h i
$ i >
4:27 PM
04/08/2012
National Board of Medical Examiners - Google Chrom e Q https://www.starttest.com
/api/5.1.1.0/ITDStart.aspx?SVC=lbfd4732-460a-454b-90b5-9985c0560007
I___

I
Exam Section 2: Item 50 o f 50 I Mark
National Board o f Medical Examiners Comprehensive Basic Science Self-Assessment
Tim e Remaining: 2 hr 46 min 27 sec
50. An 18-year-old man comes to the physician because of nausea, headache, blood
in his urine, and malaise for 3 days. Three weeks ago, he had severe pharyngiti
s that resolved spontaneously after several days without antibiotic therapy. His
blood pressure is 165/88 mm Hg. Physical examination shows mild peripheral edem
a. His serum creatinine concentration is 2.1 mg/dl_, and serum CH50 is markedly
decreased. Urinalysis shows: Protein RBC WBC RBC casts Blood cultures are negati
ve. Which of the following is the most likely causal organism? A) Streptococcus
agalactiae (group B) B) S. mutans O C) S. pneumoniae 2+ 25 50/hpf 10-20/hpf prese
nt
D) S. pyogenes (group A) O E) Viridans streptococci
National Board of Medical Examiners - Google Chrom e Q https: 7 www.starttest.co
m/api/5.1.1.0/rrDStart.aspx?SVC=lbfd4732-460a-454b-90b5-9985c0560007
Exam Section 3: Item 1 o f 50 I Mark
National Board o f Medical Examiners Comprehensive Basic Science Self-Assessment
Tim e Remaining: 4 hr 15 min 50 sec
1. A 22-year-old woman volunteers for a phase 1 clinical trial of the pharmacoki
netics of a new drug. She receives a single intravenous bolus of the drug in her
left upper extremity. Blood samples are then drawn at different times from her
right upper extremity. The concentration of the drug is measured by a specific a
ssay, and the results are shown (log of plasma concentration [C J versus time).
Which of the following best explains the two-phase results obtained? A) Normal e
limination followed by saturation of liver enzymes B) Rapid absorption followed
by binding to plasma proteins C) Rapid absorption followed by slower elimination
o D) Rapid distribution to tissues followed by normal elimination E) Rapid excr
etion by kidneys followed by metabolism in liver
Time
Lab Values
Review

Pause
C l
4:46 PM 04/08/2012
National Board of Medical Examiners - Google Chrom e Q https://www.starttest.com
/api/5.1.1.0/ITDStart.aspx?SVC=lbfd4732-460a-454b-90b5-9985c0560007
I___ I
National Board o f Medical Examiners Comprehensive Basic Science Self-Assessment
Tim e Remaining: 4 hr 12 min 39 sec
Exam Section 3: Item 2 o f 50 I Mark
2. A 3-week-old newborn is brought to the physician because of recurrent vomitin
g after feeding since birth. The mother says that her child is eager to feed eve
n after vomiting. There is no fever. Abdominal examination shows a firm, 1- to 2
-cm, mobile mass in the epigastrium to the right of the midline. If this conditi
on has a lower threshold of liability in males than in females, which of the fol
lowing relatives of this patient is at greatest risk for also developing this di
sorder? o A) Brother, if newborn is female O B) Brother, if newborn is male C) F
raternal male twin, if newborn is male O D) Sister, if newborn is female O E) Si
ster, if newborn is male
National Board of Medical Examiners - Google Chrom e Q https://www.starttest.com
/api/5.1.1.0/ITDStart.aspx?SVC=lbfd4732-460a-454b-90b5-9985c0560007
I___ I
National Board o f Medical Examiners Comprehensive Basic Science Self-Assessment
Tim e Remaining: 4 hr 11 min 21 sec
Exam Section 3: Item 3 o f 50 I Mark
3. A 2-month-old boy is brought to the physician because of failure to thrive an
d poor feeding since birth. He was recently adopted from Romania and has been fe
d cow's milk. He is below the 5th percentile for length and weight. Physical exa
mination shows jaundice, cataracts, and hepatomegaly. Serum studies show a decre
ased glucose concentration. The urine shows a positive reaction to a copper redu
ction test and a negative reaction to a test agent that contains glucose oxidase
. Deficiency of which of the following hepatic enzymes is the most likely cause
of the disorder in this patient? O A) Fructokinase B) Fructose-1,6-bisphosphate
aldolase C) Galactokinase o D) Galactose-1-phosphate uridyltransferase E) Glucos
e-6-phosphatase
National Board of Medical Examiners - Google Chrom e
Q https://www.starttest.eom/api/5.l.1.0/TTDStart.aspx?SVC=lbfd4732-460a-454b-90b
5-9985c0560007 Exam Section 3: Item 4 o f 50 H Mark National Board o f Medical E
xaminers Comprehensive Basic Science Self-Assessment
4. A 65-year-old man undergoes an operation to remove an abdominal mass. Histolo
gic examination of the mass shows a large number of blood vessels. Further analy
sis shows an increased concentration of vascular endothelial growth factor (VEGF
) in the tumor. Which of the following most likely stimulated the production of
VEGF in the mass in this patient? o A) Decreased endostatin O B) Decreased Pco2
C) Decreased Po2 D) Decreased thrombospondin E) Increased endostatin O F) Increa
sed Pco2 O G) Increased Po2 H) Increased thrombospondin
National Board of Medical Examiners - Google Chrom e Q https://www.starttest.com
/api/5.1.1.0/ITDStart.aspx?SVC=lbfd4732-460a-454b-90b5-9985c0560007
I___ I
National Board o f Medical Examiners Comprehensive Basic Science Self-Assessment
Tim e Remaining: 4 hr 8 min 35 sec
Exam Section 3: Item 5 o f 50 I Mark
5. A female newborn delivered at term is found to have a cleft palate with cleft
lip bilaterally. Physical examination shows no other abnormalities. Cranial ult
rasonography shows no abnormalities. Failure of fusion at which of the following
sites is the primary cause of this patient's cleft lip? A) Frontonasal and eye
prominences B) Frontonasal and maxillary prominences C) Mandibular process and e
xternal auditory meatus D) Mandibular prominence and secondary pharyngeal arch o
E) Maxillary and nasal prominences
[^f National Board of Medical Examiners - Google Chrom e
|
| fay r
-|
Q https://www.starttest.com/api/5.1.1.0/TTDStart.aspx?SVC=lbfd4732-460a-454b-90b
5-9985c0560007 Exam Section 3: Item 6 o f 50 H Mark National Board o f Medical E
xaminers Comprehensive Basic Science Self-Assessment Tim e Remaining: 4 hr 7 min
1 sec
6. A 57-year-old woman is brought to the emergency department because she has be
en unable to open her left eye since she awoke 2 hours ago. She has no history o
f trauma to the eye. Her vital signs are within normal limits. When the left eye
lid is raised during examination, her eye is fixed in the out position. When the
patient is asked to move her eye out, the eye intorts. An MRI of the brain show
s an aneurysm of the left posterior communicating artery. The most likely cause
of this patient's condition is compression of which of the following nerves? O A
) Abducens nerve B) Long ciliary nerve o C) Oculomotor nerve D) Ophthalmic branc
h of the trigeminal nerve E) Trochlear nerve
Previous
O
p 1
Lab Values
Review -

Pause
C l
..in
$
> <
4:56 PM 04/08/2012
LI
National Board of Medical Examiners - Google Chrom e Q https://www.starttest.com
/api/5.1.1.0/ITDStart.aspx?SVC=lbfd4732-460a-454b-90b5-9985c0560007
I__ I
National Board o f Medical Examiners Comprehensive Basic Science Self-Assessment
Tim e Remaining: 4 hr 5 min 57 sec
Exam Section 3: Item 7 o f 50 I Mark
7. A 33-year-old woman has episodes of severe paroxysmal pain in the right cheek
, lower eyelid, and upper lip. These episodes are triggered by light touch in th
e affected areas. The most likely diagnosis is neuralgia involving which of the
following nerves? A) Buccal branch of the facial B) Glossopharyngeal C) Mandibul
ar division of the trigeminal o D) Maxillary division of the trigeminal E) Ophth
almic division of the trigeminal F) Zygomatic branch of the facial
Q
https://www.starttest.com/api/5.1.1.0/TTDStart.aspx?SVC=lbfd4732-460a-454b-90b5-
9985c0560007
Exam Section 3: Item 8 o f 50 H Mark
National Board o f Medical Examiners Comprehensive Basic Science Self-Assessment
Tim e Remaining: 4 hr 3 min 17 sec
8. During an experiment, drug X is added to a muscle bath containing vascular sm
ooth muscle precontracted to 5 grams (g) tension. The effects of drug X and othe
r drugs on the force of muscle contraction (g) are shown in the table.
Standard Drugs
Drug vehicle Norepinephrine Isoproterenol Phenylephrine Which of the following d
rugs is most likely to produce effects similar to those of drug X? O A) Albutero
l O B) Atropine C) Methoxamine o D) Prazosin O E) Propranolol
Force Before Drug X (g)
5.0 15.1 0.5 13.0
Force After Drug X (g)
4.9
6.8 0.6
5.5
National Board of Medical Examiners - Google Chrom e Q https://wvw.starttest.com
/api/511M TDStart.aspx7SVC3lbfd4732-460a-454b-90b5-9985c0560007
I__ I
National Board o f Medical Examiners Comprehensive Basic Science Self-Assessment
Tim e Remaining: 3 hr 56 min 56 sec
Exam Section 3: Item 9 o f 50 0 Mark
9. A 5-year-old boy with asthma is brought to the physician because of a 3-month
history of recurrent cough, intermittent wheezing, and difficulty breathing. He
has been admitted to the hospital twice for pneumonia during the past 12 months
. He is at the 60th percentile for height and the 50th percentile for w eight Hi
s respirations are 25/min. Wheezing and crackles are heard between only the four
th to the sixth intercostal spaces on the right side of the chest. Diminished ta
ctile fremitus and dullness are present over the right anterior section of the c
hest between the 4th and 6th ribs. Results of a PPD test are negative. A lateral
chest x-ray shows a wedge-shaped density extending anteriorly and inferiorly fr
om the hilum. A CT scan of the chest is most likely to show obstruction of which
of the following bronchi? C A) Left main-stem B) Left upper lobe o C) Right low
er lobe D) Right main-stem = > E) Right middle lobe
National Board of Medical Examiners - Google Chrom e Q https://wvw.starttest.com
/api/511M TDStart.aspx7SVC3lbfd4732-460a-454b-90b5-9985c0560007
I___ I
National Board o f Medical Examiners Comprehensive Basic Science Self-Assessment
Tim e Remaining: 3 hr 56 min 19 sec
Exam Section 3: Item 10 o f 50 I Mark
10. A 6-year-old girl is brought to the physician by her mother because of swoll
en, itchy eyes, a runny nose, and sneezing for the past week. Knowledge of which
of the following components of the patient's history is likely to be most produ
ctive in establishing a diagnosis? O A) New medications o B) New pet in the hous
ehold C) Occupations of parents D) Recent illness in a family member O E) Recent
school performance
[^p National Board of Medical Examiners - Google Chrom e Q https://ww w .startte
st.eom /api/5.l.1.0/TTDStart.aspx?SVC=lbfd4732-460a-454b-90b5-9985c0560007
I'
Exam Section 3: Item 11 o f 50 I Mark
National Board o f Medical Examiners Comprehensive Basic Science Self-Assessment
Tim e Remaining: 3 hr 55 min 51 sec
11.
A 21 -year-old woman with primary pulmonary hypertension begins treatment with b
osentan. As a result, blockade of which of the following is most likely to occur
?
O A) Angiotensin II receptors B) Calcium channels o C) Endothelin receptors O D)
Production of phosphodiesterase 5 E)Voltage-gated sodium channels
Previous
Lab Values

Review
Help
-
Pause

O
.,il,
$
5:07 PM
> <
04/08/2012
National Board of Medical Examiners - Google Chrom e Q https://www.starttest.com
/api/5.1.1.0/ITDStart.aspx?SVC=lbfd4732-460a-454b-90b5-9985c0560007
I___ I
National Board o f Medical Examiners Comprehensive Basic Science Self-Assessment
Tim e Remaining: 3 hr 53 min 37 sec
Exam Section 3: Item 12 o f 50 0 Mark
12.
A 30-year-old woman comes to the physician for an initial evaluation. She has a
history of illicit intravenous drug use and is now completing a methadone mainte
nance program. Physical examination shows scars in the antecubital fossae. A che
st x-ray shows small nodules in the perihilar lung fields. A biopsy specimen fro
m the perihilar regions will most likely show small foreign particles surrounded
by which of the following? A) C) Granulation tissue Hemosiderin-laden macrophag
es
o B) Granulomatous inflammation O D) Neutrophilic abscess E)Proliferated capilla
ries
National Board of Medical Examiners - Google Chrom e Q https://www.starttest.com
/api/5.1.1.0/ITDStart.aspx?SVC=lbfd4732-460a-454b-90b5-9985c0560007
I___ I
National Board o f Medical Examiners Comprehensive Basic Science Self-Assessment
Tim e Remaining: 3 hr 51 min 34 sec
Exam Section 3: Item 13 o f 50 I Mark
13. Four children, ages 2 through 6 years, are brought to the physician by their
mother because of a 2-week history of irritability and forgetfulness. Their mot
her says, "Their behavior is the last straw. They fight all the time and copy ea
ch other's bad behavior. They are also lazy; when they're not fighting, they're slee
ping. And when I ask the older ones to do their chores, they whine and say they
can't because they have a headache or they're dizzy." She adds, "I barely have e
nough money to buy food, and I've been trying to avoid having our electricity tu
rned off by using my portable heater." Which of the following initial actions by
the physician is most appropriate? A) C) Offer the mother a behavior program wi
th the use of time-out for her children Suggest that the mother consider medicat
ion for her own stress level
O B) Review the adequacy of the children's diet o D) Assess the possibility of c
arbon monoxide toxicity O E) Call child protective services
Q
https://www.starttest.com/api/5.1.1.0/TTDStart.aspx?SVC=lbfd4732-460a-454b-90b5-
9985c0560007
Exam Section 3: Item 14 o f 50 I Mark
National Board o f Medical Examiners Comprehensive Basic Science Self-Assessment
Tim e Remaining: 3 hr 50 min 2 sec
14.
A 55-year-old woman comes to the physician because of a 3-month history of incre
ased fatigue and difficulty sleeping. She says that she feels sad and empty and
is not interested in her hobbies anymore. She has no history of major medical il
lness. She takes no medications. She does not smoke cigarettes or drink alcohol.
Physical examination and laboratory studies show no abnormalities. Treatment wi
th escitalopram is started. The next day, the patient's husband calls the physic
ian to ask about his wife's diagnosis. Which of the following is the most approp
riate response by the physician to the patient's husband?
O A) "I can tell you about your wife's diagnosis since you are her legal next of
kin." B) "I cannot discuss any patient information over the telephone, so I rec
ommend that you come to the office with your wife instead." C) "I can't disclose
your wife's diagnosis to you unless you have health care power-of-attorney righ
ts." o D) "I'm sorry, but I cannot tell you anything about your wife's diagnosis
without her permission." E) "Since I don't know the source of your wife's depre
ssion, you should discuss this with her."
National Board of Medical Examiners - Google Chrom e Q https://www.starttest.com
/api/5.1.1.0/ITDStart.aspx?SVC=lbfd4732-460a-454b-90b5-9985c0560007
I___ I
National Board o f Medical Examiners Comprehensive Basic Science Self-Assessment
Tim e Remaining: 3 hr 49 min 12 sec
Exam Section 3: Item 15 o f 50 I Mark
15. A 37-year-old woman has a 6-month history of increasing pain, stiffness, and
swelling in her hands and wrists. Ibuprofen alleviates her symptoms for brief p
eriods. Examination shows swelling of the soft tissue of the wrists and the meta
carpophalangeal and proximal interphalangeal joints bilaterally. Which of the fo
llowing is the most likely cause of these symptoms? A) Cartilage degeneration B)
Deposition of uric acid crystals O C) Infection o D) Systemic inflammatory dise
ase
0 E) Trauma
National Board of Medical Examiners - Google Chrom e Q https://www.starttest.com
/api/5.1.1.0/ITDStart.aspx?SVC=lbfd4732-460a-454b-90b5-9985c0560007
Exam Section 3: Item 16 o f 50 0 Mark
National Board o f Medical Examiners Comprehensive Basic Science Self-Assessment
Tim e Remaining: 3 hr 47 min 20 sec
16.
Protection against paralytic poliomyelitis can be achieved either by oral admini
stration of live attenuated poliovirus (Sabin) vaccine or by parenteral administ
ration of killed poliovirus (Salk) vaccine. The common features of these two vac
cines, which accounts for their efficacy, is their ability to induce which of th
e following poliovirus-specific immune responses? A) Activated CD8+ cytotoxic T
lymphocyte effectors in the circulation
O B) Activated CD8+ cytotoxic T lymphocyte effectors in the gut C) CD8+ memory T
lymphocytes D) Neutralizing antibodies in the circulation @ E) Neutralizing sec
retory IgA antibodies in the gut >
[^p National Board of Medical Examiners - Google Chrom e Q https://ww w .startte
st.eom /api/5.l.1.0/TTDStart.aspx?SVC=lbfd4732-460a-454b-90b5-9985c0560007
Exam Section 3: Item 17 o f 50 I Mark
National Board o f Medical Examiners Comprehensive Basic Science Self-Assessment
Tim e Remaining: 3 hr 46 min 10 sec
17.
A 3-year-old girl is brought to the emergency department because of nausea and v
omiting that began after she ingested a large quantity of her grandfather's "hea
rt pills." An ECG shows a third-degree atrioventricular nodal block with ventric
ular escape rhythm that resolves after the administration of atropine. Which of
the following agents was most likely ingested? A) Captopril
o B) Digoxin C) Hydrochlorothiazide D) Isosorbide dinitrate
Previous
Lab Values

Review
Help -
Pause
O
..in
$
> <
5:17 PM 04/08/2012
National Board of Medical Examiners - Google Chrom e Q https://www.starttest.com
/api/5.1.1.0/ITDStart.aspx?SVC=lbfd4732-460a-454b-90b5-9985c0560007
I___ I
National Board o f Medical Examiners Comprehensive Basic Science Self-Assessment
Tim e Remaining: 3 hr 40 min 31 sec
Exam Section 3: Item 18 o f 50 0 Mark
18. A previously unrecognized, rapidly progressive degenerative neurologic illne
ss is detected in the population of a small island in the Pacific Ocean. Autopsi
es are performed on several patients who died of the disease. Electron microscop
y of infected neurons shows spherical to conical viral particles. Brain tissue i
s then used to inoculate mice and a variety of cell lines, but no cytopathic eff
ect is observed. Further enzymatic studies show that infected cells contained RN
A-dependent DNA polymerase activity. The virus causing this infection is most cl
osely related to which of the following viruses? O A) Coxsackievirus O B) Herpes
simplex virus
m C) HIV D) Human papillomavirus O E) West Nile virus
National Board of Medical Examiners - Google Chrom e Q https://www.starttest.com
/api/5.1.1.0/ITDStart.aspx?SVC=lbfd4732-460a-454b-90b5-9985c0560007
I___ I
National Board o f Medical Examiners Comprehensive Basic Science Self-Assessment
Tim e Remaining: 3 hr 38 min 36 sec
Exam Section 3: Item 19 o f 50 I Mark
19.
During an evaluation for renal lithiasis, a 46-year-old man undergoes a series o
f imaging studies. An x-ray shows evidence of subcortical bone resorption. A dua
l energy x-ray absorptiometry scan shows decreased bone density. Laboratory stud
ies show hypercalcemia and an increased parathyroid hormone concentration. A par
athyroidectomy is recommended. During the operation, two parathyroid glands are
found posterior to the thyroid gland on the left side, but only one, which appea
rs to be the inferior, is present on the right. Because of the similarity of its
developmental origin, which of the following additional organs should be explor
ed to find the remaining parathyroid?
O A) Palatine tonsil O B) Parotid gland C) Sublingual gland D) Submandibular gla
nd o E) Thymus
Q
https://www.starttest.com/api/5.1.1.0/TTDStart.aspx?SVC=lbfd4732-460a-454b-90b5-
9985c0560007
Exam Section 3: Item 20 o f 50 0 Mark
National Board o f Medical Examiners Comprehensive Basic Science Self-Assessment
Tim e Remaining: 3 hr 34 min 36 sec
20. A 12-year-old boy is admitted to the hospital because of lethargy, hip pain,
and a temperature of 39.4C (103F). He has been hospitalized several other times b
ecause of pneumonia. His neonatal period was normal. Complete blood counts are w
ithin normal limits, and a test for HIV antibody is negative. Blood cultures gro
w Staphylococcus aureus. Serum
Next
Lab Values
Review

Pause
C l
m
Q
https://www.starttest.com/api/5.1.1.0/TTDStart.aspx?SVC=lbfd4732-460a-454b-90b5-
9985c0560007
Exam Section 3: Item 21 o f 50 H Mark
National Board o f Medical Examiners Comprehensive Basic Science Self-Assessment
Tim e Remaining: 3 hr 31 min 26 sec
21. A 5-year-old girl is brought to a new physician by her mother for an examina
tion prior to attending kindergarten. Her mother says, "My daughterhas a murmur,
but I was told that she'd grow out of it." The patient has no history of major
medical illness. She is at the 50thpercentile for height and weight. A grade 2/6
systolicmurmur is heard over the left sternal border. Cardiac catheterization s
hows:
Location
Aorta Vena cava Pulmonary artery Right atrium Left atrium Right ventricle Left v
entricle
Pressure (mm Hg)
105/60 7/2 25/9 4 8 25/4 105/8
0 2 Saturation
98% 75% 85% 75% 98% 83% 98%
The failure of which of the following during embryonic development best explains
this patient's condition? O A) Closure of the ductus venosus B) Closure of the
foramen ovale C) Development of the septum secundum o D) Fusion of the intervent
ricular septum with endocardial cushions O E) Reabsorption of the septum primum
LI
National Board of Medical Examiners - Google Chrom e Q https://wvwv.starttest.co
m/api/5.1.1.0/ITDStart.aspx?SVC=lbfd4732-460a-454b-90b5-9985c0560007
Exam Section 3: Item 22 o f 50 I Mark
National Board o f Medical Examiners Comprehensive Basic Science Self-Assessment
Tim e Remaining: 3 hr 29 min 4 sec
22. A 34-year-old woman gives birth to a newborn at 32 weeks' gestation with a s
mall placenta. The newborn is found to have multiple anomalies, including bilate
ral simian creases and a small thorax. The newborn dies 2 hours after birth. Chr
omosomal analysis shows a karyotype of 69,XXY. DNA analysis indicates that two s
ets of chromosomes are maternal in origin and one is paternal. Studies from othe
r triploid pregnancies have shown that triploidy due to two sets of paternal chr
omosomes has poorly developed embryonic tissue. Differing phenotypes dependent o
n parental origin are most compatible with which of the following genetic mechan
isms? O A) Anticipation B) Expansion of trinucleotide repeats o C) Imprinting O
D) Loss of heterozygosity O E) Pleiotropy
National Board of Medical Examiners - Google Chrom e Q https://www.starttest.com
/api/5.1.1.0/ITDStart.aspx?SVC=lbfd4732-460a-454b-90b5-9985c0560007
I__ I
National Board o f Medical Examiners Comprehensive Basic Science Self-Assessment
Tim e Remaining: 3 hr 24 min 49 sec
Exam Section 3: Item 23 o f 50 I Mark
23. A 16-year-old boy is brought to the emergency department by his mother becau
se of a 3-day history of fever, chills, and a nonproductive cough. His temperatu
re is 38.5C (101.3F). Physical examination shows several large ecchymoses and nume
rous petechiae. Laboratory studies show: Hemoglobin Hematocrit Leukocyte count P
latelet count Prothrombin time Partial thromboplastin time Plasma fibrinogen 10.
2 g/dL 33% 31,000/mm3 14,000/mm3 22 sec (INR=2.1) 51 sec 87 mg/dL (N=200-400)
A peripheral blood smear shows promyelocytes containing Auer rods. Which of the
following best explains these findings? A) Acquired factor VIII (antihemophilic)
inhibitor B) Biliary cholestasis C) Disseminated intravascular coagulation D) H
emophilia A E) Iron deficiency anemia o F) Vitamin K deficiency
I
National Board of Medical Examiners - Google Chrom e Q https://www.starttest.com
/api/5.1.1.0/ITDStart.aspx?SVC=lbfd4732-460a-454b-90b5-9985c0560007
I__ I
National Board o f Medical Examiners Comprehensive Basic Science Self-Assessment
Tim e Remaining: 3 hr 22 min 9 sec
Exam Section 3: Item 24 o f 50 0 Mark
24.
A 76-year-old woman comes to the physician because of severe back pain for the p
ast 2 weeks. She has no history of smoking. She has been taking prednisone (30 m
g/day) for rheumatoid arthritis for the past 6 months and has received hormone r
eplacement therapy with estrogen and progesterone for the past 15 years. Calcium
and vitamin D intake are adequate. X-rays of the spine show a vertebral fractur
e. Which of the following is the most likely cause of the fracture? A) Decreased
bone formation due to decreased calcium absorption
o B) Decreased bone formation due to inhibition of osteoblast differentiation C)
Increased bone resorption due to decreased calcium absorption D) Increased bone
resorption due to decreased serum parathyroid hormone concentration O E) Increa
sed bone resorption due to estrogen receptor defect
[^p National Board of Medical Examiners - Google Chrom e Q https://ww w .startte
st.eom /api/5.l.1.0/TTDStart.aspx?SVC=lbfd4732-460a-454b-90b5-9985c0560007
Exam Section 3: Item 25 o f 50 I Mark
National Board o f Medical Examiners Comprehensive Basic Science Self-Assessment
Tim e Remaining: 3 hr 21 min 3 sec
25. A 58-year-old woman comes to the physician because of a 3-week history of ab
dominal swelling and fatigue. Abdominal examination shows ascites. A left ovaria
n mass is found on pelvic examination. Microscopic examination of the ascitic fl
uid shows malignant cells. Additional malignant cells are most likely to be foun
d in which of the following groups of lymph nodes? O A) Inferior mesenteric O B)
Internal iliac o C) Para-aortic (lumbar) O D) Pararectal O E) Sacral
Previous
Lab Values

Review
Help
Pause
O
..in $ > <
5:42 PM 04/08/2012
-

National Board of Medical Examiners - Google Chrom e Q https://ww w .starttest.e


om /api/5.l.1.0/TTDStart.aspx?SVC=lbfd4732-460a-454b-90b5-9985c0560007
IrO-UgM
Exam Section 3: Item 26 o f 50 I Mark
National Board o f Medical Examiners Comprehensive Basic Science Self-Assessment
Tim e Remaining: 3 hr 20 min 37 sec
26. An 80-year-old woman develops a bleeding duodenal ulcer after a 2-month stay
in the intensive care unit. A decision is made to start the patient on a drug t
hat will adhere strongly to the ulcer crater, thereby protecting the ulcer epith
elium. Which of the following is the most appropriate pharmacotherapy? A) Calciu
m carbonate B) Magnesium hydroxide O C) Misoprostol O D) Ranitidine o E) Sucralf
ate
Previous
Lab Values

Review
Help -
Pause
O
..ill $ '< >
5:43 PM 04/08/2012
National Board of Medical Examiners - Google Chrom e Q https://www.starttest.com
/api/5.1.1.0/ITDStart.aspx?SVC=lbfd4732-460a-454b-90b5-9985c0560007
I___ I
National Board o f Medical Examiners Comprehensive Basic Science Self-Assessment
Tim e Remaining: 3 hr 19 min 20 sec
Exam Section 3: Item 27 o f 50 I Mark
27.
A 43-year-old woman comes to the physician because she is concerned about her ri
sk for developing influenza. She is a nurse, and she recently cared for a man wi
th influenza. She did not receive influenza virus vaccine this year. Physical ex
amination shows no abnormalities. A drug that blocks which of the following vira
l enzymes is most appropriate to decrease this patient's risk for developing thi
s infection?
O A) DNA polymerase B) Integrase o C) Neuraminidase D) Protease E) Reverse trans
criptase
National Board of Medical Examiners - Google Chrom e Q https://www.starttest.com
/api/5.1.1.0/ITDStart.aspx?SVC=lbfd4732-460a-454b-90b5-9985c0560007
I___ I
National Board o f Medical Examiners Comprehensive Basic Science Self-Assessment
Tim e Remaining: 3 hr 18 min 8 sec
Exam Section 3: Item 28 o f 50 I Mark
28.
A previously healthy 2-year-old girl is brought to the physician because of thre
e episodes of dark blood in her stool during the past week. Examination shows no
masses in the abdomen or rectum. Test of the stool for occult blood is positive
. A technetium 99m scan shows pooling of the radiolabel in the midabdomen. Which
of the following is the most likely diagnosis?
O A) Anal fissure B) Appendicitis C) Congenital megacolon (Hirschsprung disease)
D) Gastroesophageal reflux O E) Hemorrhoids o F) Meckel diverticulum
V > National Board of Medical Examiners - Google Chrom e ^ Q https://ww w .start
test.eom /api/5.l.1.0/TTDStart.aspx?SVC=lbfd4732-460a-454b-90b5-9985c0560007
I' C3 | dpi
Exam Section 3: Item 29 o f 50 H Mark
National Board o f Medical Examiners Comprehensive Basic Science Self-Assessment
Tim e Remaining: 3 hr 17 min 7 sec
29. A male newborn delivered at term has marked hypopigmentation with white hair
, milky white skin, and blue irides. The most likely cause of this condition is
a defective enzyme contained in which of the following organelles? A) Golgi comp
lex
O B) Lysosomes o C) Melanosomes D) Mitochondria E) Nucleus
Previous
Lab Values

Review
Help -
Pause
O
..in
$
> <
5:47 PM 04/08/2012
National Board of Medical Examiners - Google Chrom e Q https://www.starttest.com
/api/5.1.1.0/ITDStart.aspx?SVC=lbfd4732-460a-454b-90b5-9985c0560007
I___ I
National Board o f Medical Examiners Comprehensive Basic Science Self-Assessment
Tim e Remaining: 3 hr 14 min 38 sec
Exam Section 3: Item 30 o f 50 0 Mark
30. An 18-year-old man is brought to the emergency department in rural Illinois
because of fever, confusion, and right arm weakness for 8 hours. His symptoms be
gan with tingling and numbness 2 days ago. His temperature is 40C (104 F). Physica
l examination shows no other abnormalities. A lumbar puncture is done. Examinati
on of cerebrospinal fluid shows: Glucose Protein Leukocyte count Lymphocytes Ery
throcyte count 58 mg/dL 50 mg/dL 23/mm3 93% 3/mm3
An MRI of the brain shows no abnormalities. He is oriented to person but not to
place or time. During the next 4 days his confusion progresses, and he develops
hypersalivation and respiratoryfailure. He continues to deteriorate despite intu
bation. He dies four days later. Which of the following animals is the most like
ly source of this patient's infection? o A) Bat B) Cat C) Cow O D) Dog E) Sheep
Q
https://www.starttest.com/api/5.1.1.0/TTDStart.aspx?SVC=lbfd4732-460a-454b-90b5-
9985c0560007
Exam Section 3: Item 31 o f 50 0 Mark
National Board o f Medical Examiners Comprehensive Basic Science Self-Assessment
Tim e Remaining: 3 hr 12 min 32 sec
31.
A 12-year-old girl is brought to the physician because of a rash on her left but
tock for the past 2 days. The rash developed after the family returned from a 2-
week-long early summer vacation in Maine. Her vital signs are within normal limi
ts. A photograph of the lesion is shown. The likely cause of this patient's infe
ction is taxonomically and morphologically most similar to the infectious agent
of which of the following conditions? A) Bacillary angiomatosis B) Chancroid C)
Leptospirosis
C D) Lymphogranuloma venereum o E )Q fever
LI
Q
https://www.starttest.com/api/5.1.1.0/TTDStart.aspx?SVC=lbfd4732-460a-454b-90b5-
9985c0560007
Exam Section 3: Item 32 o f 50 H Mark
National Board o f Medical Examiners Comprehensive Basic Science Self-Assessment
Tim e Remaining: 3 hr 9 min 8 sec
32. A 15-month-old white boy is brought to the physician by his parents because
of recurrent pneumonia since the age of 2 months and failure to thrive for 1 yea
r. His stools are frequent and greasy. He has a 2-year-old cousin with a similar
condition; his two older sisters are healthy. He is at the 10th percentile for
length and below the 5th percentile for weight. Bilateral rhonchi are heard in b
oth lungs. Examination of the stool shows an increased fat concentration. A ches
t x-ray shows increased bronchovascular markings bilaterally. There is no consan
guinity. He is individual III-2 in the pedigree shown. Which of the following is
the most likely explanation for the pattern of inheritance in this patient's fa
mily? A) Autosomal dominant with variable expressivity B) Autosomal dominant wit
h variable penetrance o C) Autosomal recessive with high heterozygote frequency
D) Autosomal recessive with low heterozygote frequency O E)X-linked rare gene
O
+
Unaffected fem ale U naffected m ale A ffected fem ale A ffected m ale
LI
National Board of Medical Examiners - Google Chrom e Q https://www.starttest.com
/api/5.1.1.0/ITDStart.aspx?SVC=lbfd4732-460a-454b-90b5-9985c0560007
I__ I
National Board o f Medical Examiners Comprehensive Basic Science Self-Assessment
Tim e Remaining: 3 hr 8 min 15 sec
Exam Section 3: Item 33 o f 50 I Mark
33.
A 50-year-old woman develops night sweats and a feeling of intense heat over the
trunk and face. Pelvic examination shows atrophic vaginal mucosa but no other a
bnormalities. A Pap smear shows an increase in parabasal epithelial cells with n
o dysplasia. These symptoms are most likely due to decreased production of which
of the following? A) Follicle-stimulating hormone by the pituitary B) Gonadotro
pin-releasing hormone by the hypothalamus C) Human chorionic gonadotropin produc
tion by thecal cells
O D) Luteinizing hormone by the pituitary o E) Steroid hormones by ovarian folli
cles
National Board of Medical Examiners - Google Chrom e Q https://www.starttest.com
/api/5.1.1.0/ITDStart.aspx?SVC=lbfd4732-460a-454b-90b5-9985c0560007
I__ I
National Board o f Medical Examiners Comprehensive Basic Science Self-Assessment
Tim e Remaining: 3 hr 6 min 57 sec
Exam Section 3: Item 34 o f 50 I Mark
34.
An investigator is studying oxygen transport in experimental animals. It is foun
d that Po2in the renal vein is relatively high compared with venous Po2from most
other organs. Which of the following best explains this finding? A) Counter-cur
rent multiplication in the kidneys allows them to use less oxygen for ion transp
ort than other organs
O B) Po2 in the renal artery is greater than Po2 in other arteries o C) The rati
o of oxygen consumption to blood flow is lower in the kidneys than in other orga
ns D) Renal blood flow is a smaller fraction of the cardiac output than blood fl
ow in other organs O E) Tissue Po2 is greater in every region of the kidneys
[^p National Board of Medical Examiners - Google Chrom e Q https://ww w .startte
st.eom /api/5.l.1.0/TTDStart.aspx?SVC=lbfd4732-460a-454b-90b5-9985c0560007
Exam Section 3: Item 35 o f 50 I Mark
National Board o f Medical Examiners Comprehensive Basic Science Self-Assessment
Tim e Remaining: 3 hr 6 min 8 sec
35. A 52-year-old woman with type 2 diabetes mellitus has chronic renal failure.
She takes no medications other than glipizide. Creatinine clearance is 20% of n
ormal. Which of the following sets of laboratory findings in serum is most likel
y in this patient?
Ca2+
A) B) C) D) E) F) * *
Phosphate i
Parathyroid Hormone I
i i T i
I + I
I
Previous
Lab Values

Review
Help
-
Pause

O
.,il,
$
5:58 PM
> <
04/08/2012
National Board of Medical Examiners - Google Chrom e Q https://www.starttest.com
/api/5.1.1.0/ITDStart.aspx?SVC=lbfd4732-460a-454b-90b5-9985c0560007
I___ I
National Board o f Medical Examiners Comprehensive Basic Science Self-Assessment
Tim e Remaining: 3 hr 4 min 39 sec
Exam Section 3: Item 36 o f 50 I Mark
36. A previously healthy 20-year-old primigravid woman at 12 weeks' gestation co
mes to the physician 2 days after she noticed a painful swelling on the right si
de of her chest. Physical examination shows a 2 x 2-cm skin-colored mass with a
nipple in the center on the chest wall just inferior to the right breast. If a b
iopsy specimen of the mass were obtained, it would most likely show a predominan
ce of which of the following cell types? o A) Epithelial cells O B) Lymphocytes
C) Macrophages D) Mast cells O E) Neutrophils
National Board of Medical Examiners - Google Chrom e Q https://www.starttest.com
/api/5.1.1.0/ITDStart.aspx?SVC=lbfd4732-460a-454b-90b5-9985c0560007
Exam Section 3: Item 37 o f 50 I Mark
National Board o f Medical Examiners Comprehensive Basic Science Self-Assessment
Tim e Remaining: 3 hr 3 min 35 sec
37. A 28-year-old woman comes to the physician because of a 1-day history of sho
rtness of breath and a 1-week history of fever, malaise, and a dry, nonproductiv
e cough. Two months ago, she received an allogeneic bone marrow transplant that
was complicated by acute graft-versus-host disease. Her temperature is 38.3C (101F
) and respirations are 42/min. There is no rash. Generalized crackles are heard
on auscultation of the chest. An x-ray of the chest shows bilateral interstitial
infiltrates. A photomicrograph of tissue obtained at lung biopsy is shown. The
most likely cause of her condition is infection with which of the following viru
ses? A) Adenovirus o B) Cytomegalovirus C) Enterovirus D) Influenza virus E) Mea
sles virus F) Papillomavirus G) Parainfluenza virus H) Respiratory syncytial vir
us I) Rhinovirus J) Varicella-zoster virus
Q
https://www.starttest.com/api/5.1.1.0/TTDStart.aspx?SVC=lbfd4732-460a-454b-90b5-
9985c0560007
Exam Section 3: Item 38 o f 50 H Mark
National Board o f Medical Examiners Comprehensive Basic Science Self-Assessment
Tim e Remaining: 3 hr 0 min 35 sec
38. A 45-year-old woman comes to the physician because of a 2-month history of f
atigue, nausea, and generalized bone pain. She has had a 2-kg (4.4-lb) weight lo
ss during this period because of loss of appetite. She has a 4-month history of
renal insufficiency. Her blood pressure is 170/100 mm Hg. Physical examination s
hows pallor and 2+ pitting edema of the feet. Laboratory studies show anemia. He
r serum urea nitrogen concentration is 55 mg/dL, and serum creatinine concentrat
ion is 4 mg/dL. Bone x-rays show widened osteoid seams and subperiosteal erosion
s. Which of the following additional sets of serum findings is most likely in th
is patient?
Calcium
m A) B) C) D) E) F) G) * * *
Phosphate
Parathyroid Hormone +
i + I I
1,25-Dihydroxycholecalciferol
i i
^ .
i * *
^ ,
I I I
^ .
i
i
1
[^p National Board of Medical Examiners - Google Chrom e Q https://ww w .startte
st.eom /api/5.l.1.0/TTDStart.aspx?SVC=lbfd4732-460a-454b-90b5-9985c0560007
Exam Section 3: Item 39 o f 50 I Mark
National Board o f Medical Examiners Comprehensive Basic Science Self-Assessment
Tim e Remaining: 2 hr 58 min 58 sec
39. A21-year-old man comes to the physician because of a 2-day history of fever,
cough, and shortness of breath. He appears mildly ill. His temperature is 38.4C
(101.1F), and respirations are 18/min. Diffuse crackles are heard on auscultation
. A cold hemagglutinin test result is positive. A chest x-ray shows increased in
terstitial markings. The infectious agent is most likely susceptible to which of
the following drugs? O A) Amoxicillin o B) Azithromycin O C) Ceftriaxone O D)Ge
ntamicin E) Piperacillin
Previous
Lab Values

Review
Help
Pause
O
..in $ > <
6:05 PM 04/08/2012
-

National Board of Medical Examiners - Google Chrom e Q https://www.starttest.com


/api/5.1.1.0/ITDStart.aspx?SVC=lbfd4732-460a-454b-90b5-9985c0560007
I__ I
National Board o f Medical Examiners Comprehensive Basic Science Self-Assessment
Tim e Remaining: 2 hr 57 min 5 sec
Exam Section 3: Item 40 o f 50 I Mark
40. In an experimental animal, blood flow to a portion of the ventricular myocar
dium is 80% restricted by placing a ligature along a branch of the coronary arte
ry. Thirty minutes later, blood flow is restored. Thirty minutes after the resto
ration of blood flow, the affected tissue is isolated for analysis. Histologic e
xamination of the transiently ischemic region shows an increase in myocardial ce
ll diameter compared with normal myocardium from the same heart and blebbing of
the sarcolemma. The swelling of these cells is most directly attributable to whi
ch of the following changes? O A) Clumping of nuclear chromatin B) Decreased int
racellular pH o C) Decreased sarcoplasmic ATP O D) Detachment of ribosomes O E)
Exit of creatine kinase O F) Increased glycolysis G) Increased protein synthesis
O H) Rupture of lysosomes
National Board of Medical Examiners - Google Chrom e Q https://www.starttest.com
/api/5.1.1.0/ITDStart.aspx?SVC=lbfd4732-460a-454b-90b5-9985c0560007
I___ I
National Board o f Medical Examiners Comprehensive Basic Science Self-Assessment
Tim e Remaining: 2 hr 55 min 22 sec
Exam Section 3: Item 41 o f 50 I Mark
41. A 6-month-old boy is brought to the physician by his mother because of a 10-
week history of cold-like symptoms. The mother says that he tastes salty when sh
e kisses him, and he has large, foul-smelling stools. There is no family history
of a similar condition. He is at the 10th percentile for length and 3rd percent
ile for weight. Physical examination shows reduction in subcutaneous fat. Coarse
rhonchi are heard bilaterally. Molecular analysis shows deletion of the Phe res
idue at position 508 in the cystic fibrosis transmembrane conductance regulator
(CFTR). Which of the following most likely occurred in this patient as a result
of this gene mutation? o A) Altered CFTR folding O B) 3-Amyloid configuration of
CFTR C) Decreased surfactant protein synthesis O D) Increased CFTR conductance
O E) Increased CFTR in the membrane
[^p National Board of Medical Examiners - Google Chrom e Q https://ww w .startte
st.eom /api/5.l.1.0/TTDStart.aspx?SVC=lbfd4732-460a-454b-90b5-9985c0560007
Exam Section 3: Item 42 o f 50 I Mark
National Board o f Medical Examiners Comprehensive Basic Science Self-Assessment
Tim e Remaining: 2 hr 49 min 55 sec
42. In an epidemiologic study of workers in the aniline dye industry, 500 worker
s with bladder cancer and 200 workers without bladder cancer are selected. The i
nvestigators obtain a history of exposure to aniline dye in both groups of worke
rs. Among workers with bladder cancer, 250 have a history of exposure to aniline
dyes; among the 200 workers without bladder cancer, 50 have a history of exposu
re. Which of the following is the odds ratio for the exposure variable? A) 0.33
B) 0.5 C) 1.0 D) 1.5 E) 2.0 o F) 3.0
Previous
Lab Values

Review
Help
Pause
O
..in $ > <
6:23 PM 04/08/2012
-

National Board of Medical Examiners - Google Chrom e Q https://www.starttest.com


/api/5.1.1.0/ITDStart.aspx?SVC=lbfd4732-460a-454b-90b5-9985c0560007
I__ I
National Board o f Medical Examiners Comprehensive Basic Science Self-Assessment
Tim e Remaining: 2 hr 48 min 0 sec
Exam Section 3: Item 43 o f 50 I Mark
43. A 38-year-old man is scheduled to undergo a gastric reduction operation for
treatment of morbid obesity. During the procedure to reduce stomach volume, a pl
astic band will be placed around the upper portion of the stomach. To encircle t
he stomach, the band will pass through which of the following structures? A) Hep
atoduodenal ligament
o B) Lesser omentum O C) Phrenicocolic ligament D) Splenorenal ligament O E) Tra
nsverse mesocolon
National Board of Medical Examiners - Google Chrom e Q https://www.starttest.com
/api/5.1.1.0/ITDStart.aspx?SVC=lbfd4732-460a-454b-90b5-9985c0560007
I___ I
National Board o f Medical Examiners Comprehensive Basic Science Self-Assessment
Tim e Remaining: 2 hr 44 min 58 sec
Exam Section 3: Item 44 o f 50 I Mark
44. A 37-year-old woman comes to the physician for a follow-up examination. She
has a 5-year history of cardiovascular disease and underwent an operation to ins
ert two prosthetic valves 6 weeks ago. X-rays of the chest are shown. Which of t
he following valves was replaced by the prosthetic valve indicated by the arrows
? A) Aortic B) Mitral o C) Pulmonic D) Tricuspid
National Board of Medical Examiners - Google Chrom e Q https://ww w .starttest.e
om /api/5.l.1.0/TTDStart.aspx?SVC=lbfd4732-460a-454b-90b5-9985c0560007
IrO-UgM
Exam Section 3: Item 45 o f 50 I Mark
National Board o f Medical Examiners Comprehensive Basic Science Self-Assessment
Tim e Remaining: 2 hr 44 min 6 sec
45.
A previously healthy 4-year-old boy is brought to the physician because of pain
in his right ear and irritability for the past 2 days. His temperature is 38.2C (
100.8F). Examination of his right ear shows a red, opaque, bulging tympanic membr
ane. Which of the following is the most likely causal organism?
O A) Escherichia coli B) Mycobacterium tuberculosis C) Neisseria meningitidis D)
Staphylococcus aureus E) Streptococcus pneumoniae
Previous
O
Next
O
Lab Values
Review

Help
Pause

-
..h i
$ i >
6:29 PM 04/08/2012
National Board of Medical Examiners - Google Chrom e g https://www.starttest.eom
/api/5.1.1.0/TTDStart.aspx?SVC=lbfd4732-460a-454b-90b5-9985c0560007
Exam Section 3: Item 46 o f 50 I Mark
National Board o f Medical Examiners Comprehensive Basic Science Self-Assessment
Tim e Remaining: 2 hr 41 min 23 sec
46. A 32-year-old woman is found to have panic disorder with agoraphobia. A drug
is prescribed that activates benzodiazepine binding sites on the Y-aminobutyric
acidA (GABA a) receptor. This drug is most likely which of the following? o A)
Alprazolam O B) Buspirone O C) Flumazenil D) Hydroxyzine O E) Ramelteon
National Board of Medical Examiners - Google Chrom e Q https://www.starttest.com
/api/5.1.1.0/ITDStart.aspx?SVC=lbfd4732-460a-454b-90b5-9985c0560007
I___ I
National Board o f Medical Examiners Comprehensive Basic Science Self-Assessment
Tim e Remaining: 2 hr 39 min 52 sec
Exam Section 3: Item 47 o f 50 I Mark
47.
A 24-year old man has had a purulent urethral discharge and dysuria for the past
2 days. Microscopic examination of a Gram stain of the urethral exudate shows g
ram-negative intracellular diplococci. He is treated with a standard dose of cef
triaxone. The symptoms disappear in 2 days but a mucoid discharge appears 10 day
s after treatment. Which of the following is the most likely explanation for the
reappearance of symptoms? A) Ceftriaxone does not penetrate host cells
o B) The patient was initially infected with both Neisseria gonorrhoeae and Chla
mydia trachomatis C) The patient was reinfected by an untreated sexual partner D
) The patient's strain of Neisseria gonorrhoeae developed resistance to ceftriax
one
Q
https://www.starttest.com/api/5.1.1.0/TTDStart.aspx?SVC=lbfd4732-460a-454b-90b5-
9985c0560007
Exam Section 3: Item 48 o f 50 H Mark
National Board o f Medical Examiners Comprehensive Basic Science Self-Assessment
Tim e Remaining: 2 hr 38 min 37 sec
48. A 45-year-old man is brought to the emergency department 30 minutes after su
staining a gunshot wound to his abdomen. His blood pressure is 70/40 mm Hg. Phys
ical examination shows pallor and diaphoresis. There is an entry wound in the mi
d-left region of the abdomen. His hematocrit is 20%. A CT scan of the abdomen is
shown. An exploratory laparotomy shows that the abdominal cavity is filled with
blood. Which of the following organs is most likely damaged in this patient? O
A) Descending colon B) Left adrenal gland C) Left kidney o D) Spleen E) Tail of
the pancreas
National Board of Medical Examiners - Google Chrom e Q https://www.starttest.com
/api/5.1.1.0/ITDStart.aspx?SVC=lbfd4732-460a-454b-90b5-9985c0560007
Exam Section 3: Item 49 o f 50 I Mark
National Board o f Medical Examiners Comprehensive Basic Science Self-Assessment
Tim e Remaining: 2 hr 35 min 9 sec
49. A 31-year-old man comes to the physician because of chronic pain and decreas
ed range of motion in the left knee. He runs 5 miles daily. Evaluation 2 months
ago showed no structural abnormalities. The physician recommended physical thera
py, use of nonsteroidal anti-inflammatory drugs (NSAIDs), and decreased running.
The patient stopped physical therapy after 1week, rarely took NSAIDs, continued
to run, and missed several appointments. Today he speaks with some sadness abou
t the inadequacy of his pain control. Which of the following best describes the
personality trait exhibited by this behavior? o A) Borderline C B) Dependent C)
Histrionic C D) Obsessive-compulsive C E) Paranoid F) Passive-aggressive
National Board of Medical Examiners - Google Chrom e Q https://www.starttest.com
/api/5.1.1.0/ITDStart.aspx?SVC=lbfd4732-460a-454b-90b5-9985c0560007
I___ I
National Board o f Medical Examiners Comprehensive Basic Science Self-Assessment
Tim e Remaining: 2 hr 33 min 38 sec
Exam Section 3: Item 50 o f 50 I Mark
50. A 17-year-old African American boy is brought to the emergency department be
cause of a 5-hour history of fever, progressive petechial rash, headache, and ri
gors. He was treated successfully for bacteremia due to Neisseria meningitidis 2
years ago. His temperature is 39.8C (103.6F), pulse is 120/min, respirations are
30/min, and blood pressure is 85/56 mm Hg. Physical examination shows evolving p
etechiae and purpura on the lower extremities. Treatment with empiric broad-spec
trum antibiotics is begun. Twelve hours later, blood culture grows N. meningitid
is. This patient should be examined for which of the following conditions? A) Ag
ammaglobulinemia B) Chronic granulomatous disease o C) Complement deficiency D)
HIV infection O E) Sickle cell disease
National Board of Medical Examiners - Google Chrom e Q https://wvw.starttest.com
/api/511M TDStart.aspx7SVC3lbfd4732-460a-454b-90b5-9985c0560007
Exam Section 4: Item 1 o f 50 I Mark
National Board o f Medical Examiners Comprehensive Basic Science Self-Assessment
Tim e Remaining: 4 hr 17 min 38 sec
1. In immunocompromised patients, the case definition of a positive tuberculin s
kin test is changed from 10 mm of induration to 5 mm of induration. This change
in case definition will have which of the following effects on incidence and pre
valence of a positive tuberculin skin test?
Incidence
A) B) C) D) E)
o
Prevalence
T
<> <>
I I
T
i
p 1
Lab Values Review
6:47 PM 04/08/2012
National Board of Medical Examiners - Google Chrom e Q https://www.starttest.com
/api/5.1.1.0/ITDStart.aspx?SVC=lbfd4732-460a-454b-90b5-9985c0560007
I___ I
National Board o f Medical Examiners Comprehensive Basic Science Self-Assessment
Tim e Remaining: 4 hr 15 min 52 sec
Exam Section 4: Item 2 o f 50 I Mark
2. A full-term 1-day-old male newborn undergoes hearing screening. The newborn h
as no visible malformations or abnormalities. Results of otoacoustic emissions t
esting are abnormal. A subsequent diagnostic brain stem auditory evoked response
is also abnormal. Which of the following is the most compelling reason for this
newborn screening program? A) Although congenital hearing loss occurs infrequen
tly, screening is cost-effective B) Early diagnosis and treatment of hearing los
s will prevent delay in motor development o C) Identification and treatment of h
earing loss before the age of 6 months will allow for better prognosis of speech
and language development D) Newborn screening allows for more time to prepare h
earing aids so the newborn can be fitted with them when he or she reaches the ag
e of 1 year E) Newborn screening will identify those children who will require a
cochlear implant after the age of 5 years
Q
https://www.starttest.com/api/5.1.1.0/TTDStart.aspx?SVC=lbfd4732-460a-454b-90b5-
9985c0560007
Exam Section 4: Item 3 o f 50 H Mark
National Board o f Medical Examiners Comprehensive Basic Science Self-Assessment
Tim e Remaining: 4 hr 15 min 26 sec
3. A 12-month-old girl has a large extended family and is cared for by numerous
relatives. According to her mother, the child does not spend much time on the fl
oor because "somebody always seems to be carrying her around." Under these circu
mstances, this child is most likely to have problems with which of the following
aspects of her development? O A) Cognitive B) Fine motor o C) Gross motor D)Lan
guage E) Social
National Board of Medical Examiners - Google Chrom e Q https://www.starttest.com
/api/5.1.1.0/ITDStart.aspx?SVC=lbfd4732-460a-454b-90b5-9985c0560007
I__ I
National Board o f Medical Examiners Comprehensive Basic Science Self-Assessment
Tim e Remaining: 4 hr 13 min 31 sec
Exam Section 4: Item 4 o f 50 I Mark
4. A 20-year-old man comes to the physician because of a 1-year history of progr
essive numbness of his arms. He recently burned his right thumb while cooking, b
ut he felt no pain. Sensation to pain and temperature is decreased throughout bo
th upper extremities and across the shoulders to the neck. Sensation is intact e
lsewhere. The most likely cause of his condition is a lesion in which of the fol
lowing spinal cord structures? o A) Anterior white commissure O B) Dorsal column
C) Intermediolateral cell column D) Lateral corticospinal tract E) Spinothalami
c tract
National Board of Medical Examiners - Google Chrom e Q https://www.starttest.com
/api/5.1.1.0/ITDStart.aspx?SVC=lbfd4732-460a-454b-90b5-9985c0560007
I__ I
National Board o f Medical Examiners Comprehensive Basic Science Self-Assessment
Tim e Remaining: 4 hr 12 min 9 sec
Exam Section 4: Item 5 o f 50 I Mark
5. A 52-year-old man is brought to the emergency department 30 minutes after the
onset of acute chest pain radiating to his back, shortness of breath, and weakn
ess. He has a history of scoliosis. He is 208 cm (6 ft 10 in) tall and weighs 91
kg (200 lb); BMI is 21 kg/m2. His blood pressure is 90/60 mm Hg. Physical exami
nation shows absence of pulses in the right upper extremity. Which of the follow
ing is the most likely diagnosis? o A) Aortic dissection B) Aortic occlusion C)
Aortic stenosis D) Coarctation of the aorta E) Rheumatic aortitis
National Board of Medical Examiners - Google Chrom e Q https://www.starttest.com
/api/5.1.1.0/ITDStart.aspx?SVC=lbfd4732-460a-454b-90b5-9985c0560007
I__ I
National Board o f Medical Examiners Comprehensive Basic Science Self-Assessment
Tim e Remaining: 4 hr 10 min 16 sec
Exam Section 4: Item 6 o f 50 I Mark
6. A 24-year-old woman is brought to the emergency department 30 minutes after h
er father found her unresponsive on her bedroom floor. Her father says that the
patient had been despondent for 2 weeks because of the ending of a long-term rel
ationship. She had recently seen a physician seeking help for her emotional stat
e. On arrival, she is unresponsive to painful stimuli. Her temperature is 35.7C (
96.2F), pulse is 52/min, respirations are 5/min and shallow, and blood pressure i
s 85/55 mm Hg. Physical examination shows pinpoint pupils that are reactive to l
ight. Neurologic examination shows no focal findings. This patient most likely h
as toxicity from which of the following? O A) Carbon monoxide B) Ethylene glycol
C) Methanol o D) Opiates E) Selective serotonin reuptake inhibitors F) Tricycli
c antidepressants
Q
https://www.starttest.com/api/5.1.1.0/TTDStart.aspx?SVC=lbfd4732-460a-454b-90b5-
9985c0560007
Exam Section 4: Item 7 o f 50 H Mark
National Board o f Medical Examiners Comprehensive Basic Science Self-Assessment
Tim e Remaining: 4 hr 8 min 34 sec
7. A 79-year-old woman with a history of severe dementia, type 2 diabetes mellit
us, and hypertension is brought to the emergency department by her son because o
f chest pain and agitation for 4 hours. She had smoked 2 packs of cigarettes dai
ly until the age of 70 years, when she quit. The son is not aware of the patient
's medication regimen. Her pulse is 120/min, respirations are 32/min, and blood
pressure is 180/100 mm Hg. Crackles are heard in both lung bases on auscultation
of the chest. Cardiac examination shows a systolic ejection murmur at the apex
and a regular rhythm. An ECG shows ST-segment elevation in the anterolateral lea
ds. A chest x-ray shows a mildly enlarged cardiac silhouette. Which of the follo
wing is the most likely diagnosis?
= > A) Acute coronary syndrome o B) Acute pericarditis C) Bilateral pneumonia C
D) Cerebrovascular event 0 E) Pulmonary embolism
National Board of Medical Examiners - Google Chrom e Q https://www.starttest.com
/api/5.1.1.0/ITDStart.aspx?SVC=lbfd4732-460a-454b-90b5-9985c0560007
I__ I
National Board o f Medical Examiners Comprehensive Basic Science Self-Assessment
Tim e Remaining: 4 hr 7 min 54 sec
Exam Section 4: Item 8 o f 50 I Mark
8. A 27-year-old woman comes to the physician because of a 5-day history of a mo
derate vaginal discharge. Physical examination shows a thin, white, vaginal disc
harge that has a foul smelling fishy odor. Vaginal pH is 6. A Pap smear shows clu
mps of bacteria on the squamous cells. There are no leukocytes. Culture of the d
ischarge is most likely to show an increased amount of which of the following? A
) Candida albicans B) Chlamydia trachomatis
C) Gardnerella vaginalis
D) Mycoplasma genitalium E) Trichomonas vaginalis
National Board of Medical Examiners - Google Chrom e Q https://www.starttest.com
/api/5.1.1.0/ITDStart.aspx?SVC=lbfd4732-460a-454b-90b5-9985c0560007
I__ I
National Board o f Medical Examiners Comprehensive Basic Science Self-Assessment
Tim e Remaining: 4 hr 6 min 38 sec
Exam Section 4: Item 9 o f 50 I Mark
9. A 35-year-old man who is an undocumented immigrant is diagnosed with tubercul
osis. Despite feeling ill, he did not previously seek medical care because he co
uld not afford to lose time at work. He pleads with the physician not to report
the diagnosis to anyone because he is afraid he will be deported. After empathiz
ing with the patient, it is most appropriate for the physician to take which of
the following actions? A) Do not report the case and insist that the patient wea
r a mask at all times B) Do not report the case as long as the patient agrees to
isolate himself while being treated C) Do not report the case because that woul
d violate the patient's right to privacy and confidentiality D) Do not report th
e case, but begin contacting the patient's family members and work site contacts
directly to arrange for evaluation E) Report the case to the health department
but assure the patient that no other individuals will be notified o F) Report th
e case to the health department to ensure that family members and work site cont
acts are identified and evaluated G) Report the case to the immigration and natu
ralization service
National Board of Medical Examiners - Google Chrom e Q https://www.starttest.com
/api/5.1.1.0/ITDStart.aspx?SVC=lbfd4732-460a-454b-90b5-9985c0560007
I__ I
National Board o f Medical Examiners Comprehensive Basic Science Self-Assessment
Tim e Remaining: 4 hr 5 min 55 sec
Exam Section 4: Item 10 o f 50 I Mark
10.
An outbreak of multidrug-resistant Escherichia coli in an intensive care unit is
being investigated. The determinants of antibiotic resistance are thought to be
carried on a plasmid that has been transferred among different bacterial strain
s. Which of the following observations from in vitro studies best supports this
hypothesis?
O A) Lysogeny must precede transfer B) Transfer is susceptible to DNase C) Trans
fer requires a bacteriophage o D) Transfer requires cell-to-cell contact E)Trans
fer requires transformation competent recipient strain
National Board of Medical Examiners - Google Chrom e Q https://wvw.starttest.com
/api/511M TDStart.aspx7SVC3lbfd4732-460a-454b-90b5-9985c0560007
I___ I
National Board o f Medical Examiners Comprehensive Basic Science Self-Assessment
Tim e Remaining: 4 hr 0 min 45 sec
Exam Section 4: Item 11 o f 50 0 Mark
11. A 45-year-old woman comes to the physician because of a 3-month history of d
ecreased energy and impaired concentration. She has had tingling in both hands,
constipation, and a 2-kg (4-lb) weight gain over the past 2 months. She appears
apathetic. Her skin has a doughy consistency and her hair is coarse. Tapping the
median nerves over the volar surface of the wrist elicits paresthesias in the i
ndex and middle fingers. The relaxation phase of the muscle stretch reflexes is
slowed. Which of the following is the most likely diagnosis? O A) Addison diseas
e B) Hyperparathyroidism C) Hypoparathyroidism D) Hypopituitarism o E) Hypothyro
idism
National Board of Medical Examiners - Google Chrom e Q https: 7 www.starttest.co
m/api/5.1.1.0/ITDStart.aspx?SVC=lbfd4732-460a-454b-90b5-9985c0560007
I___ I
National Board o f Medical Examiners Comprehensive Basic Science Self-Assessment
Tim e Remaining: 3 hr 59 min 19 sec
Exam Section 4: Item 12 o f 50 I Mark
12. In a clinical trial, a 44-year-old woman with end-stage renal disease underg
oes kidney transplantation in combination with bone marrow transplantation from
the same donor. Monitoring of the allograft post-transplantation shows good func
tion despite the lack of treatment with immunosuppressive drugs. The transplante
d bone marrow is most likely inducing which of the following immune phenomena in
this recipient? O A) Co-stimulation B) Hypersensitivity O C) Senescence O D) Su
rveillance o E) Tolerance
National Board of Medical Examiners - Google Chrom e Q https://www.starttest.com
/api/5.1.1.0/ITDStart.aspx?SVC=lbfd4732-460a-454b-90b5-9985c0560007
Exam Section 4: Item 13 o f 50 I Mark
National Board o f Medical Examiners Comprehensive Basic Science Self-Assessment
Tim e Remaining: 3 hr 53 min 49 sec
13. A 20-year-old man is brought to the emergency department 4 hours after recei
ving a gunshot wound to his forearm during a hunting accident. An exploratory op
eration shows that the tendon of the flexor carpi radialis is severed, and there
is a comminuted fracture of the distal radius; all other structures are intact.
Due to the extent of the injury, an attempt to suture the severed ends of the c
arpi radialis tendon is unsuccessful. The tendon of which of the following muscl
es is most likely to be used to surgically bridge the gap between the severed en
ds of the injured tendon because functional loss would be insignificant? A) Flex
or carpi ulnaris C B) Flexor digitorum profundus C) Flexor digitorum s u p e rfi
c ia l o E) Pronator teres
C f e D) Palmaris longus
National Board of Medical Examiners - Google Chrom e Q https://www.starttest.com
/api/5.1.1.0/ITDStart.aspx?SVC=lbfd4732-460a-454b-90b5-9985c0560007
I__ I
National Board o f Medical Examiners Comprehensive Basic Science Self-Assessment
Tim e Remaining: 3 hr 52 min 45 sec
Exam Section 4: Item 14 o f 50 I Mark
14.
A cough suppressant (Drug X) is compared with codeine with the following results
:
Characteristic
Cough suppression Analgesia Constipation Abuse potential
Codeine
++++ +++ +++ ++
Drug X
+++ minimal minimal +
Drug X most closely resembles which of the following drugs? A) Chlorpheniramine
o B) Dextromethorphan C) Guaifenesin D) Meperidine O E) Phenylephrine
National Board of Medical Examiners - Google Chrom e Q https://www.starttest.com
/api/5.1.1.0/ITDStart.aspx?SVC=lbfd4732-460a-454b-90b5-9985c0560007
I__ I
National Board o f Medical Examiners Comprehensive Basic Science Self-Assessment
Tim e Remaining: 3 hr 51 min 49 sec
Exam Section 4: Item 15 o f 50 I Mark
15. An investigative team is studying the physiologic mechanisms that explain in
creased exercise performance during 2 weeks at high altitude (10,000 feet above
sea level) in experimental animals. These experiments are designed to distinguis
h between increases in arterial oxygen delivery to exercising muscle and unloadi
ng oxygen from blood into the tissues. Which of the following factors would be e
xpected to increase for the data to support increased unloading oxygen into the
tissues with acclimatization to high altitude? A) Arterial pH
0
B) Arterial Po2 o C) 2,3-Bisphosphoglycerate
0 D) Hematocrit 0 E) Net hemoglobin
L^p National Board of Medical Examiners - Google Chrom e
0
https://www.starttest.com/api/5.1.1.0/ITDStart.aspx?SVC=lbfd4732-460a-454b-90b5-
9985c0560007
Exam Section 4: Item 16 o f 50 H Mark
National Board o f Medical Examiners Comprehensive Basic Science Self-Assessment
Tim e Remaining: 3 hr 50 min 34 sec
16. A 32-year-old woman with restless legs syndrome comes to the physician for a
follow-up examination. She says that she now has symptoms three to four times w
eekly. A drug is prescribed that acts as an agonist on dopamine (D2) receptors.
This drug is most likely which of the following? O A) Baclofen O B)Gabapentin O
C)Levodopa o D) Ropinirole 0; E) Selegiline
Previous
O
Lab Values
Review

Pause
7:15 PM 04/08/2012

[^p National Board of Medical Examiners - Google Chrom e


I'
Q
https://www.starttest.com/api/5.1.1.0/TTDStart.aspx?SVC=lbfd4732-460a-454b-90b5-
9985c0560007
Exam Section 4: Item 17 o f 50 0 Mark
National Board o f Medical Examiners Comprehensive Basic Science Self-Assessment
Tim e Remaining: 3 hr 48 min 32 sec
17. A previously healthy 6-month-old boy develops fever and severe paroxysmal ba
rking coughing. He has not had any immunizations. His leukocyte count is 30,000/
mm3 (70% lymphocytes). Neutrophil chemotaxis and oxidative metabolism are defect
ive due to increased activity of which of the following enzymes? o A) Adenylyl c
yclase B) Myeloperoxidase
0 C) NADPH oxidase 0 D) Phospholipase C 0 E) Protein kinase C
Previous
O
O &
Next
Lab Values
Review

Help
Pause

H r% r T O 1 i
-

..III
$ i >
7:17 PM
04/08/2012
National Board of Medical Examiners - Google Chrom e Q https://www.starttest.com
/api/5.1.1.0/ITDStart.aspx?SVC=lbfd4732-460a-454b-90b5-9985c0560007
I___ I
National Board o f Medical Examiners Comprehensive Basic Science Self-Assessment
Tim e Remaining: 3 hr 46 min 34 sec
Exam Section 4: Item 18 o f 50 0 Mark
18.
During a study of the response of renal tubular cells to ischemic injury, the bl
ood supply to the kidneys in experimental animals is interrupted for variable le
ngths of time. The renal tubular cells then are examined by light and electron m
icroscopy. The presence of which of the following features would definitively in
dicate irreversible injury to the renal tubular cells? A) Blunting of microvilli
B) Cellular swelling C) Disaggregation of ribosomes
o D) Disruption of the plasma membrane E) Loosening of intercellular attachments
National Board of Medical Examiners - Google Chrom e Q https://www.starttest.com
/api/5.1.1.0/ITDStart.aspx?SVC=lbfd4732-460a-454b-90b5-9985c0560007
I__ I
National Board o f Medical Examiners Comprehensive Basic Science Self-Assessment
Tim e Remaining: 3 hr 45 min 11 sec
Exam Section 4: Item 19 o f 50 I Mark
19. A 4-year-old girl is brought to the emergency department 30 minutes after su
staining a burn on her right index finger when she touched a hot stove. Examinat
ion of the finger shows a tense, clear-fluid-filled blister. Before the blister
resolves, epithelium forms at the base of the blister. Which of the following ce
lls is involved in the re-epithelization of the blister in this patient? A) Basa
l layer keratinocytes C) Epithelial Langerhans cells
c >
C B) Dermal dendritic cells o D) Fibroblasts 0 E) Macrophages
National Board of Medical Examiners - Google Chrom e
4
National Board o f Medical Examiners Comprehensive Basic Science Self-Assessment
Tim e Remaining: 3 hr 44 min 5 sec
Q
h ttp s://w w w .sta rtte st.e o m /a p i/5 .l.1.0/TTDStart.aspx?SVC=lbfd4732-46
0a-454b-90b5-9985c0560007
Exam Section 4: Item 20 o f 50 I Mark
20.
A female newborn dies at the age of 12 days. A photograph of a section of the br
ain as seen at autopsy is shown. The newborn was most likely affected by which o
f the following? A) Arteriovenous malformation B) Germinal matrix hemorrhage
o C) Hyperbilirubinemia D) Hypoxia-ischemia E) Transtentorial herniation
Previous
O
O &
Next
Lab Values
Review

Help
Pause

H r% r T O 1 i
-

..hi
$ i >
7:21 PM
04/08/2012
T
National Board of Medical Examiners - Google Chrom e Q https://www.starttest.com
/api/5.1.1.0/ITDStart.aspx?SVC=lbfd4732-460a-454b-90b5-9985c0560007
Exam Section 4: Item 21 o f 50 I Mark
National Board o f Medical Examiners Comprehensive Basic Science Self-Assessment
Tim e Remaining: 3 hr 43 min 37 sec
21. An obese 45-year-old woman has a 16-year history of intermittent episodes of
an epigastric burning sensation. Use of antacids usually relieves her symptoms.
Esophagogastroscopy shows velvety patches in the mucosa of the distal esophagus
. Which of the following best explains the process shown in the photomicrograph
of a biopsy specimen from one of the patches?
A) Atrophy B) Hyperplasia O C) Hypertrophy o D) Metaplasia E) Steatosis
National Board of Medical Examiners - Google Chrom e Q https://www.starttest.com
/api/5.1.1.0/ITDStart.aspx?SVC=lbfd4732-460a-454b-90b5-9985c0560007
I___ I
National Board o f Medical Examiners Comprehensive Basic Science Self-Assessment
Tim e Remaining: 3 hr 40 min 40 sec
Exam Section 4: Item 22 o f 50 I Mark
22.
A 28-year-old man is brought to the emergency department because of blurred visi
on, fatigue, and severe thirst 20 minutes after being found wandering along a de
serted farming road. He reports that his car had run out of gas 3 hours ago, and
he had walked approximately 7 miles toward his destination without water or sha
de from the intense summer heat. His current body weight is 92% of the value lis
ted on his driver's license. His temperature is 39.9C (103.8F), pulse is 105/min,
and blood pressure is 95/60 mm Hg. Physical examination shows pale, dry skin. He
is occasionally incoherent. He is placed supine on a bed, and intravenous fluid
s are administered. Within 60 minutes, the patient is able to take fluids by mou
th and provides additional information about his next of kin before falling asle
ep. At the time when the man was found on the road, which of the following physi
ologic parameters would have been decreased compared with normal values? A) Plas
ma renin activity B) Renal (3-adrenergic receptor stimulation
o C) Renal arteriolar perfusion pressure D) Renal sympathetic nerve activation E
) Serum angiotensin II concentration
National Board of Medical Examiners - Google Chrom e Q https://www.starttest.com
/api/5.1.1.0/ITDStart.aspx?SVC=lbfd4732-460a-454b-90b5-9985c0560007
I___ I
National Board o f Medical Examiners Comprehensive Basic Science Self-Assessment
Tim e Remaining: 3 hr 38 min 13 sec
Exam Section 4: Item 23 o f 50 I Mark
23.
Six weeks after undergoing an open carpal tunnel release operation, a 44-year-ol
d woman comes to the physician because of new symptoms of numbness of her right
hand. When she is asked about the exact location of these symptoms, she points t
o the area over the right thenar eminence. Percussion of the area between the fl
exor carpi radialis and palmaris longus tendons at the distal palmar wrist creas
e produces a painful shock-like sensation radiating into the affected area of th
e palm. An intraoperative nerve injury is suspected. Which of the following nerv
es is most likely injured in this patient? A) Dorsal sensory branch of the ulnar
nerve B) Lateral cutaneous nerve of the forearm
S
C) Palmar cutaneous branch of the median nerve o D) Recurrent motor branch of th
e median nerve E) Sensory branch of the radial nerve
National Board of Medical Examiners - Google Chrom e Q https://www.starttest.com
/api/5.1.1.0/ITDStart.aspx?SVC=lbfd4732-460a-454b-90b5-9985c0560007
I___ I
National Board o f Medical Examiners Comprehensive Basic Science Self-Assessment
Tim e Remaining: 3 hr 36 min 19 sec
Exam Section 4: Item 24 o f 50 I Mark
24. An 18-year-old man develops a fever 6 days after returning to Wisconsin from
a trip to India. He did not take antimalarial prophylaxis. A peripheral blood s
mear shows numerous intraerythrocytic forms of Plasmodium vivax. He is treated w
ith chloroquine, and his symptoms resolve. Six months later, he again develops f
ever, and his blood smear again shows intraerythrocytic forms of P. vivax. He ha
s not traveled since his trip to India. Which of the following is the most likel
y explanation for the recurrence of symptoms in this patient? o A) Activation of
exoerythrocytic forms O B) Dual infection with P. falciparum C) Poor compliance
with antimicrobial therapy O D) Reinfection O E) Resistance to chloroquine
National Board of Medical Examiners - Google Chrom e Q https://www.starttest.com
/api/5.1.1.0/ITDStart.aspx?SVC=lbfd4732-460a-454b-90b5-9985c0560007
I___ I
National Board o f Medical Examiners Comprehensive Basic Science Self-Assessment
Tim e Remaining: 3 hr 33 min 57 sec
Exam Section 4: Item 25 o f 50 0 Mark
25.
A 35-year-old man comes to the physician for a follow-up examination. He has a 1
0-year history of bradykinesia, tremors, and rigidity consistent with Parkinson
disease. His parents are in their late 50s and have no symptoms of Parkinson dis
ease. He has no siblings. Genetic analysis shows two loss-of-function mutations
in the parkin gene, which encodes a ubiquitin protein ligase. Which of the follo
wing pathologic processes is most likely to occur in this patient as a result of
these mutations? A) Absence of neuronal proteasomes
o B) Accumulation of aggregated protein within the cytoplasm of neurons C) Exces
s dopaminergic neurons in basal ganglia D) Increased protein aggregates within t
he nucleus of neurons E) Proliferation of pigmented neurons in the substantia ni
gra
National Board of Medical Examiners - Google Chrom e Q https://www.starttest.com
/api/5.1.1.0/ITDStart.aspx?SVC=lbfd4732-460a-454b-90b5-9985c0560007
I___ I
National Board o f Medical Examiners Comprehensive Basic Science Self-Assessment
Tim e Remaining: 3 hr 30 min 6 sec
Exam Section 4: Item 26 o f 50 I Mark
26. A 42-year-old woman comes for a routine follow-up examination. She has an 8-
year history of hypertension treated with a thiazide diuretic. She reports that
she tires easily with exertion because of her weight but is otherwise asymptomat
ic. She drinks one to two glasses of wine weekly. There is no family history of
liver disease. She is 163 cm (5 ft 4 in) tall and weighs 77 kg (170 lb); BMI is
29 kg/m2. Her blood pressure is 140/90 mm Hg. Examination shows no other abnorma
lities except for truncal obesity with no abdominal striae. Serum studies show:
Urea nitrogen Glucose Creatinine Total cholesterol HDL-cholesterol Triglycerides
AST ALT 19mg/dL 117mg/dL 1 mg/dL 227 mg/dL 32 mg/dL 347 mg/dL 87 U/L 85 U/L
Which of the following is the most likely cause of this patient's increased seru
m liver enzyme activity? O A) Alcoholic hepatitis B) Hemochromatosis C) Hepatiti
s A o D) Nonalcoholic steatohepatitis E) Porphyria cutanea tarda
f Q
R a tio n a l Board of Medical Examiners - Google Chrom e https://www.starttest.
eom/api/5.1.1.0/TTDStart.aspx?SVC=lbfd4732-460a-454b-90b5-9985c0560007
Exam Section 4: Item 27 o f 50 I Mark
National Board o f Medical Examiners Comprehensive Basic Science Self-Assessment
Tim e Remaining: 3 hr 29 min 22 sec
27.
A 25-year-old man who is seropositive for HIV has had progressive apathy and dys
arthria for 3 months. Examination shows a homonymous hemianopia. An MRI shows mu
ltiple enhancing lesions in the cerebral cortex. Which of the following is the m
ost likely diagnosis?
A) Colloid cyst B) Craniopharyngioma O C) Ependymoma D) Ganglioneuroma o E) Lymp
homa F) Medulloblastoma G) Meningioma H) Retinoblastoma I) Schwannoma
National Board of Medical Examiners - Google Chrom e Q https://www.starttest.com
/api/5.1.1.0/ITDStart.aspx?SVC=lbfd4732-460a-454b-90b5-9985c0560007
I___ I
National Board o f Medical Examiners Comprehensive Basic Science Self-Assessment
Tim e Remaining: 3 hr 27 min 57 sec
Exam Section 4: Item 28 o f 50 I Mark
28. A 32-year-old African American man comes to the physician for a routine heal
th maintenance examination. He feels well and has no history of serious illness.
His brother received the diagnosis of anemia at the age of 20 years. The patien
t's pulse is 76/min, respirations are 16/min, and blood pressure is 128/68 mm Hg
. Examination shows no abnormalities. Laboratory studies show: Hemoglobin Mean c
orpuscular volume Red cell distribution width Serum Ferritin Iron 11 g/dL 70 pm3
13% (N=13% -15%) 200 ng/mL 150 pg/dL
A blood smear shows hypochromic microcytic erythrocytes with occasional target c
ells. Which of the following is most likely to confirm the diagnosis? o A) Hemog
lobin electrophoresis B) Measurement of serum thyroid-stimulating hormone concen
tration C) Measurement of serum vitamin B 12 (cobalamin) concentration O D) Colo
noscopy O E) Bone marrow biopsy
[^p National Board of Medical Examiners - Google Chrom e
I'
Q
h ttp s://w w w .sta rtte st.e o m /a p i/5 .l.1.0/TTDStart.aspx?SVC=lbfd4732-46
0a-454b-90b5-9985c0560007
Exam Section 4: Item 29 o f 50 I Mark
National Board o f Medical Examiners Comprehensive Basic Science Self-Assessment
Tim e Remaining: 3 hr 26 min 45 sec
29. An investigator conducts a study of gastrointestinal drugs that can be used
preoperatively to minimize the risk for regurgitation and pulmonary aspiration o
f gastric contents during an operation. It is found that one of these drugs prom
otes gastric emptying and relieves nausea and vomiting. This drug is most likely
which of the following? A) Dronabinol
o B) Metoclopramide O C) Misoprostol 0 D) Ondansetron 0 E) Scopolamine
Previous
O
O &
Next
Lab Values
Review

Help -
Pause ..in $ > < 7:39 PM
04/08/2012

H r% r T O 1 i
National Board of Medical Examiners - Google Chrom e Q https://www.starttest.com
/api/5.1.1.0/ITDStart.aspx?SVC=lbfd4732-460a-454b-90b5-9985c0560007
I__ I
National Board o f Medical Examiners Comprehensive Basic Science Self-Assessment
Tim e Remaining: 3 hr 24 min 40 sec
Exam Section 4: Item 30 o f 50 I Mark
30. A study is conducted to assess whether a new antiviral drug is effective in
treating symptomatic outbreaks in patients with herpes simplex virus (HSV) infec
tions. A total of 1000 patients from a community with recurrent HSV infections a
re enrolled and randomized: 500 patients receive the new drug, and 500 patients
receive placebo. Results show that the average duration of recurrent herpes simp
lex outbreaks in the new drug group is 1 week, compared with 2 weeks in the plac
ebo group. Which of the following outcomes regarding symptomatic HSV infections
is most likely in the greater population if all affected patients in the communi
ty are treated with the new drug? O A) Incidence will decrease by half O B) Inci
dence will double o C) Prevalence will decrease by half O D) Prevalence will dou
ble
National Board of Medical Examiners - Google Chrom e Q https://www.starttest.com
/api/5.1.1.0/ITDStart.aspx?SVC=lbfd4732-460a-454b-90b5-9985c0560007
Exam Section 4: Item 31 o f 50 I Mark
National Board o f Medical Examiners Comprehensive Basic Science Self-Assessment
Tim e Remaining: 3 hr 23 min 0 sec
31.
During an experiment, the US11 gene product of cytomegalovirus is expressed cons
titutively after stable transfection in an experimental tumor cell line. It is f
ound that this gene product causes translocation of nascent class I MHC molecule
s from the endoplasmic reticulum into the cytosol. Which of the following is mos
t likely to occur regarding the class I MHC products? A) Association with invari
ant chain
C B) Binding of peptides from the endocytic pathway = > C) Degradation by the pr
oteasome D) Formation of class I MHC/class II MHC hybrid molecules o E) Interact
ion with the T-lymphocyte receptor
[^p National Board of Medical Examiners - Google Chrom e
I'
Q
https://www.starttest.com/api/5.1.1.0/TTDStart.aspx?SVC=lbfd4732-460a-454b-90b5-
9985c0560007
Exam Section 4: Item 32 o f 50 I Mark
National Board o f Medical Examiners Comprehensive Basic Science Self-Assessment
Tim e Remaining: 3 hr 21 min 45 sec
32. A 58-year-old man who has smoked 2 packs of cigarettes daily for 30 years ha
s severe chronic bronchitis. Which of the following arterial blood gas values wi
th the patient breathing room air are most compatible with his condition? pH A)
B) C) D) E) 7.30 7.32 7.42 7.50 7.52 P c o 2(mm Hg) 18 65 40 55 20 P o 2(mm 95 6
0 95 90 60
Previous
Lab Values

Review
Help
Pause
O
.,il, $ > <
7:44 PM 04/08/2012
-

National Board of Medical Examiners - Google Chrom e Q https://www.starttest.com


/api/5.1.1.0/ITDStart.aspx?SVC=lbfd4732-460a-454b-90b5-9985c0560007
Exam Section 4: Item 33 o f 50 0 Mark
National Board o f Medical Examiners Comprehensive Basic Science Self-Assessment
Tim e Remaining: 3 hr 20 min 6 sec
33. During a study, a healthy 35-year-old woman and a 35-year-old woman with Cro
hn disease are given an oral solution of radiolabeled lactulose. The appearance
of radioactivity in their urine is monitored over the next 24 hours. Urinary exc
retion of radioactivity in the patient with Crohn disease is much greater than t
hat of the healthy subject. This finding is most likely due to inflammation-asso
ciated changes in the integrity of which of the following epithelial structures?
o A) Adherens junctions (zonulae adherentes) C B) Basement membrane C) Desmosom
es (maculae adherentes) D) Gap junctions = > E) Tight junctions (zonulae occlude
ntes)
National Board of Medical Examiners - Google Chrom e Q https://www.starttest.com
/api/5.1.1.0/ITDStart.aspx?SVC=lbfd4732-460a-454b-90b5-9985c0560007
I___ I
National Board o f Medical Examiners Comprehensive Basic Science Self-Assessment
Tim e Remaining: 3 hr 19 min 0 sec
Exam Section 4: Item 34 o f 50 I Mark
34.
A 40-year-old man comes to the physician for a routine health maintenance examin
ation. There is a strong family history of myocardial infarctions occurring at t
he ages of 40 to 50 years. Physical examination shows no abnormalities. Serum st
udies show: Cholesterol, total HDL-cholesterol LDL-cholesterol Triglycerides inc
reased normal increased normal
Following treatment with lovastatin, his serum total cholesterol and serum LDL-c
holesterol concentrations decrease. The beneficial effect of lovastatin most lik
ely occurs as a result of which of the following mechanisms? A) Decreases absorp
tion of bile acids in intestine B) Increases hydrolysis of triglycerides o C) In
creases LDL-receptors D) Increases lipoprotein lipase activity O E) Inhibits the
secretion of apo B-48
[^f National Board of Medical Examiners - Google Chrom e
Q https://www.starttest.com/api/5.1.1.0/TTDStart.aspx?SVC=lbfd4732-460a-454b-90b
5-9985c0560007 Exam Section 4: Item 35 o f 50 H Mark National Board o f Medical
Examiners Comprehensive Basic Science Self-Assessment Tim e Remaining: 3 hr 17 m
in 6 sec
35. A lesion of which of the following lettered structures results in loss of th
e pupillary light reflex in the left eye when light is shone into either eye?
Previous
O
Next
Lab Values
Review

Pause
C l
7:49 PM 04/08/2012
m
National Board of Medical Examiners - Google Chrom e Q https://www.starttest.com
/api/5.1.1.0/ITDStart.aspx?SVC=lbfd4732-460a-454b-90b5-9985c0560007
I__ I
National Board o f Medical Examiners Comprehensive Basic Science Self-Assessment
Tim e Remaining: 3 hr 13 min 35 sec
Exam Section 4: Item 36 o f 50 I Mark
36.
After an emergency landing in the wilderness, a 25-year-old pilot goes 3 days wi
thout food until she is rescued. Adipocytes play an important role in maintainin
g homeostasis in this woman because of which of the following effects? A) Glucag
on activates glycerol utilization as a source of carbon for gluconeogenesis by a
dipocytes B) Glucagon activates glycogen mobilization to form free glucose
o C) Glucagon activates hormone-sensitive lipase D) Insulin activates fatty acid
synthesis E) Insulin activates hepatic lipoprotein lipase
National Board of Medical Examiners - Google Chrom e Q https://www.starttest.com
/api/5.1.1.0/ITDStart.aspx?SVC=lbfd4732-460a-454b-90b5-9985c0560007
I__ I
National Board o f Medical Examiners Comprehensive Basic Science Self-Assessment
Tim e Remaining: 3 hr 10 min 54 sec
Exam Section 4: Item 37 o f 50 0 Mark
37.
A 16-year-old boy who frequently goes fishing is brought to the physician becaus
e of a 1-month history of a lesion on the front of his right shoulder. He has bl
ond hair and fair skin. Physical examination shows an 0.8-cm ulcerated, slightly
raised nodule that is variably brown to black. Examination of a biopsy specimen
of the nodule confirms a superficially invasive melanoma with features of regre
ssion. Which of the following best explains the lesion's regression in this pati
ent? A) Antibody-dependent cellular reaction
o B) Antibody-mediated cellular dysfunction O C) Arthus reaction D) Complement-
and Fc receptor-mediated response E) T-lymphocyte-mediated cytotoxicity
National Board of Medical Examiners - Google Chrom e Q https://www.starttest.com
/api/5.1.1.0/ITDStart.aspx?SVC=lbfd4732-460a-454b-90b5-9985c0560007
I___ I
National Board o f Medical Examiners Comprehensive Basic Science Self-Assessment
Tim e Remaining: 3 hr 9 min 38 sec
Exam Section 4: Item 38 o f 50 I Mark
38.
A 38-year-old man comes to the physician because of a 3-week history of rectal p
ain and bleeding that is worse when he defecates. He had similar symptoms 1 year
ago. He is afebrile. Anoscopy shows the findings in the photograph. This patien
t most likely has which of the following conditions?
O A) Anal fistula B) Colorectal carcinoma C) Condylomata acuminata o D) Hemorrho
ids O E) Perianal abscess
National Board of Medical Examiners - Google Chrom e Q https://www.starttest.com
/api/5.1.1.0/ITDStart.aspx?SVC=lbfd4732-460a-454b-90b5-9985c0560007
I___ I
National Board o f Medical Examiners Comprehensive Basic Science Self-Assessment
Tim e Remaining: 3 hr 6 min 44 sec
Exam Section 4: Item 39 o f 50 I Mark
39.
A 34-year-old man with an 8-year history of HIV infection comes to the physician
for a follow-up examination. His condition has remained stable with highly acti
ve antiretroviral therapy consisting of one nonnucleoside reverse transcriptase
inhibitor (NNRTI) and two nucleoside reverse transcriptase inhibitors (NRTIS). B
lockade of which of the following best explains the beneficial effects of NNRTI
therapy in this patient?
O A) Apoptosis of HIV-infected cells B) D) Cleavage of viral envelope precursor
glycoproteins Integrase functions in host cells O C) Entry of HIV virus into hos
t T lymphocytes o E) Replication of the HIV viral genome in host cells
Q
https://www.starttest.com/api/5.1.1.0/TTDStart.aspx?SVC=lbfd4732-460a-454b-90b5-
9985c0560007
Exam Section 4: Item 40 o f 50 H Mark
National Board o f Medical Examiners Comprehensive Basic Science Self-Assessment
Tim e Remaining: 3 hr 5 min 26 sec
40. A 50-year-old man comes to the physician for a follow-up examination. He has
poorly controlled hypertension despite three-drug therapy. He is 183 cm (6 ft)
tall and weighs 90 kg (198 lb); BMI is 27 kg/m2. His pulse is 72/min and regular
, and blood pressure is 180/102 mm Hg. An S 4and an abdominal bruit are heard on
auscultation. Serum studies show a sodium concentration of 142 mEq/L, potassium
concentration of 2.4 mEq/L, and creatinine concentration of 1 mg/dl_. MR angiog
raphy of the abdomen shows narrowing of the right renal artery. Which of the fol
lowing sets of hormone findings is most likely in this patient?
Plasma Renin
m
A) B) C) D) E) F) G) H) I) * r Normal Normal Normal
Serum Aldosterone
* normal r normal
normal
National Board of Medical Examiners - Google Chrom e Q https://www.starttest.com
/api/5.1.1.0/ITDStart.aspx?SVC=lbfd4732-460a-454b-90b5-9985c0560007
I__ I
National Board o f Medical Examiners Comprehensive Basic Science Self-Assessment
Tim e Remaining: 3 hr 3 min 47 sec
Exam Section 4: Item 41 o f 50 I Mark
41. A 35-year-old woman with leiomyomata uteri undergoes an experimental treatme
nt that involves instillation of an embolizing agent directly into the vessels t
hat feed the leiomyomata. During this procedure, in order to reach the involved
vessels, which of the following is the most direct course of the catheter after
entering the femoral artery? A) Aorta -> gonadal artery -> uterine artery o B) E
xternal iliac artery -> internal iliac artery -> uterine artery O C) External il
iac artery -> superior gluteal artery -> uterine artery O D) Internal iliac arte
ry -> inferior vesical artery -> uterine artery O E) Internal iliac artery -> su
perior vesical artery -> uterine artery
National Board of Medical Examiners - Google Chrom e Q https://www.starttest.com
/api/5.1.1.0/ITDStart.aspx?SVC=lbfd4732-460a-454b-90b5-9985c0560007
I___ I
National Board o f Medical Examiners Comprehensive Basic Science Self-Assessment
Tim e Remaining: 3 hr 2 min 48 sec
Exam Section 4: Item 42 o f 50 I Mark
42. A 40-year-old man with type 2 diabetes mellitus comes to the physician for a
follow-up examination. He currently takes no medications, and his condition is
being treated by diet. He has a sedentary lifestyle. He is 178 cm (5 ft 10 in) t
all and weighs 104 kg (230 lb); BMI is33 kg/m2.Physical examination shows no oth
er abnormalities. His fasting serum hemoglobin A 1c is 9%. Treatment with glybur
ide is started. This drug is most likely to have a beneficial effect in thispati
ent by whichof the following mechanisms? o A) Blocking an ATP-sensitive potassiu
m channel in the pancreatic beta cells B) Blocking the conversion of sucrose to
glucose in the liver C) Inhibiting a cytochrome P450 enzyme that degrades insuli
n D) Interfering with the absorption of glucose from the gastrointestinal tract
E) Stimulating insulin uptake by the liver by promoting the formation of caveola
e in the membrane
National Board of Medical Examiners - Google Chrom e
Q
h ttp s://w w w .sta rtte st.e o m /a p i/5 .l.1.0/TTDStart.aspx?SVC=lbfd4732-46
0a-454b-90b5-9985c0560007
Exam Section 4: Item 43 o f 50 I Mark
National Board o f Medical Examiners Comprehensive Basic Science Self-Assessment
Tim e Remaining: 3 hr 1 min 57 sec
>y %. '4
*f
. 'v #
. < *# I* .
, * * *> <
i r
, . . . i
v * * .
' K
V ?
43. A 24-year-old woman comes to the physician because of a 2-week history of ho
arseness and difficulty swallowing. She has a 14-year history of mild persistent
asthma successfully treated with daily administration of inhalational fluticaso
ne and albuterol as needed. A photograph of the tongue is shown. Microscopic exa
mination of scrapings from these areas shows budding yeast. Which of the followi
ng is the most appropriate pharmacotherapy for this patient's oral infection? A)
Doxycycline
O B) Flucytosine o C) Nystatin O D) Penicillin G benzathine E)Terbinafine
Q
https://www.starttest.com/api/5.1.1.0/TTDStart.aspx?SVC=lbfd4732-460a-454b-90b5-
9985c0560007
Exam Section 4: Item 44 o f 50 H Mark
National Board o f Medical Examiners Comprehensive Basic Science Self-Assessment
Tim e Remaining: 3 hr 0 min 44 sec
44. A 60-year-old man comes to the physician for a routine health maintenance ex
amination. Physical examination shows no abnormalities. Urinalysis shows: pH Spe
cific gravity Blood Glucose Protein 6.0 1.018 3+ negative 1+
Microscopic examination of the urine shows atypical cells. A CT scan of the abdo
men discloses a lesion of the right kidney as shown in the photograph. Which of
the following compounds is the most significant predisposing risk factor for thi
s patient's condition? A) Arsenic B) Beryllium o C) 2-Naphthylamine D) Nickel E)
Vinyl chloride
National Board of Medical Examiners - Google Chrom e Q https://www.starttest.com
/api/5.1.1.0/ITDStart.aspx?SVC=lbfd4732-460a-454b-90b5-9985c0560007
I__ I
National Board o f Medical Examiners Comprehensive Basic Science Self-Assessment
Tim e Remaining: 2 hr 59 min 51 sec
Exam Section 4: Item 45 o f 50 I Mark
45. A 59-year-old man comes to the physician because of a 3-week history of dull
left-sided chest pain, shortness of breath, weakness, and nonproductive cough.
He also has had an 11 -kg (25-lb) weight loss during the past 3 months. He has w
orked as an insulation installer for 30 years. He is 185 cm (6 ft 1 in) tall and
now weighs 70 kg (155 lb); BMI is 20 kg/m2. His respirations are 26/min. Breath
sounds are diminished on the left side. A chest x-ray is shown. A biopsy specim
en shows an anaplastic, biphasic neoplasm that expresses calretinin and cytokera
tin but not carcinoembryonic antigen. Which of the following additional structur
es is most likely in this patient's lung tissue? O A) Birbeck granules B) Dense
core secretory granules o C) Ferruginous bodies O D) Signet ring cells E) Silica
crystals
i
' [^N ational Board of Medical Examiners - Google Chrom e
Q
https://www.starttest.com/api/5.1.1.0/TTDStart.aspx?SVC=lbfd4732-460a-454b-90b5-
9985c0560007
Exam Section 4: Item 46 o f 50 I Mark
National Board o f Medical Examiners Comprehensive Basic Science Self-Assessment
Tim e Remaining: 2 hr 57 min 52 sec
46.
An investigation is conducted to examine a new drug (Drug X) for the treatment o
f asthma. The drug is found to increase cAMP in smooth muscle cells. The mechani
sm of action of Drug X most closely resembles that of which of the following dru
gs?
o A) Albuterol B) Ipratropium C) Montelukast O D) Nedocromil O E)Omalizumab
Previous
Lab Values

Review
Help
-
Pause

O
.,il,
$
8:08 PM
> <
04/08/2012
National Board of Medical Examiners - Google Chrom e Q https://www.starttest.com
/api/5.1.1.0/ITDStart.aspx?SVC=lbfd4732-460a-454b-90b5-9985c0560007
I__ I
National Board o f Medical Examiners Comprehensive Basic Science Self-Assessment
Tim e Remaining: 2 hr 56 min 32 sec
Exam Section 4: Item 47 o f 50 I Mark
47. A 52-year-old man comes to the physician for a follow-up visit 2 weeks after
being released from the hospital following a myocardial infarction. Since disch
arge, he has not had chest pain, dyspnea on exertion, paroxysmal nocturnal dyspn
ea, orthopnea, or edema. He has smoked 2 packs of cigarettes daily for the past
35 years, except for the week he was in the hospital. When the physician asks hi
m about smoking cessation, he says he is not interested. Which of the following
is the most appropriate next step to help the patient stop smoking? o A) Ask the
patient if he has any concerns about smoking and his health O B) Give him a pre
scription for bupropion C) Give him a prescription for nicotine replacement ther
apy D) Have the patient set a quit date to stop smoking E) Tell the patient he w
ill have another myocardial infarction if he continues to smoke F) Tell the pati
ent he will need to find another physician if he does not quit smoking
National Board of Medical Examiners - Google Chrom e Q https://www.starttest.com
/api/5.1.1.0/ITDStart.aspx?SVC=lbfd4732-460a-454b-90b5-9985c0560007
I__ I
National Board o f Medical Examiners Comprehensive Basic Science Self-Assessment
Tim e Remaining: 2 hr 56 min 1 sec
Exam Section 4: Item 48 o f 50 I Mark
48.
A 67-year-old man comes to the physician because of tingling and numbness of his
hands for 10 days. He has non-Hodgkin lymphoma treated with cyclophosphamide, d
oxorubicin, prednisone, rituximab, and vincristine. Sensation to pinprick is dec
reased over the hands and feet. Which of the following drugs in this patient's m
edication regimen is the most likely cause of these findings? A) Cyclophosphamid
e B) Doxorubicin
C) Prednisone D) Rituximab o E) Vincristine
National Board of Medical Examiners - Google Chrom e Q https://www.starttest.com
/api/5.1.1.0/ITDStart.aspx?SVC=lbfd4732-460a-454b-90b5-9985c0560007
Exam Section 4: Item 49 o f 50 I Mark
National Board o f Medical Examiners Comprehensive Basic Science Self-Assessment
Tim e Remaining: 2 hr 54 min 56 sec
49.
A 60-year-old man develops pain, erythema, and swelling of the right great toe.
Serum uric acid concentration is three times normal. Which of the following find
ings is most common in patients with this condition? A) Absence of aminotransfer
ase B) Absence of glucose 6-phosphatase C) Absence of glutathione peroxidase
o D) Absence of hypoxanthine guanine phosphoribosyltransferase = > E) No specifi
c enzyme or renal defect

Das könnte Ihnen auch gefallen